Discussion:
Wer kann helfen, ein merkwürdiges Verhältnis zu interpretieren?
(zu alt für eine Antwort)
Ganzhinterseher
2020-06-06 10:23:47 UTC
Permalink
Zur Erinnerung: Georg Cantor nummeriert alle positiven rationalen Zahlen in seiner berühmten Folge:

1/1, 1/2, 2/1, 1/3, 3/1, 1/4, 2/3, 3/2, 4/1, 1/5, 5/1, 1/6, ... .

Bisher hat niemand den Grenzwert des Verhältnisses der Anzahl der positiven rationalen Zahlen aus dem ersten und dem n-ten Einheitsintervall, also aus (0, 1] und (n, n+1], ausrechnen können, nicht einmal das Verhältnis aus (0, 1] und (1000, 1001]. Aber das ist sicher größer als 100.

Kann wenigstens jemand die Bedeutung dieses Verhältnisses interpretieren?

Gruß, WM
jvr
2020-06-06 11:18:08 UTC
Permalink
Post by Ganzhinterseher
1/1, 1/2, 2/1, 1/3, 3/1, 1/4, 2/3, 3/2, 4/1, 1/5, 5/1, 1/6, ... .
Bisher hat niemand den Grenzwert des Verhältnisses der Anzahl der positiven rationalen Zahlen aus dem ersten und dem n-ten Einheitsintervall, also aus (0, 1] und (n, n+1], ausrechnen können, nicht einmal das Verhältnis aus (0, 1] und (1000, 1001]. Aber das ist sicher größer als 100.
Kann wenigstens jemand die Bedeutung dieses Verhältnisses interpretieren?
Gruß, WM
Sie halten diese triviale Aufgabe also für schwierig?
Kinder, da ist etwas faul! So ein kleines Hirn und so ein großes Maul!
Ganzhinterseher
2020-06-06 11:27:09 UTC
Permalink
Post by jvr
Post by Ganzhinterseher
1/1, 1/2, 2/1, 1/3, 3/1, 1/4, 2/3, 3/2, 4/1, 1/5, 5/1, 1/6, ... .
Bisher hat niemand den Grenzwert des Verhältnisses der Anzahl der positiven rationalen Zahlen aus dem ersten und dem n-ten Einheitsintervall, also aus (0, 1] und (n, n+1], ausrechnen können, nicht einmal das Verhältnis aus (0, 1] und (1000, 1001]. Aber das ist sicher größer als 100.
Kann wenigstens jemand die Bedeutung dieses Verhältnisses interpretieren?
Sie halten diese triviale Aufgabe also für schwierig?
Wollen sehen, was zusammenkommt.

Gruß, WM
Ganzhinterseher
2020-06-13 12:31:22 UTC
Permalink
Post by jvr
Post by Ganzhinterseher
1/1, 1/2, 2/1, 1/3, 3/1, 1/4, 2/3, 3/2, 4/1, 1/5, 5/1, 1/6, ... .
Bisher hat niemand den Grenzwert des Verhältnisses der Anzahl der positiven rationalen Zahlen aus dem ersten und dem n-ten Einheitsintervall, also aus (0, 1] und (n, n+1], ausrechnen können, nicht einmal das Verhältnis aus (0, 1] und (1000, 1001]. Aber das ist sicher größer als 100.
Kann wenigstens jemand die Bedeutung dieses Verhältnisses interpretieren?
Sie halten diese triviale Aufgabe also für schwierig?
Kinder, da ist etwas faul! So ein kleines Hirn und so ein großes Maul!
Seit dem 6. 6. ist eine Woche verflossen. Kommt da noch was aus dem großen Maul?

Gruß, WM
jvr
2020-06-13 16:00:01 UTC
Permalink
Post by Ganzhinterseher
Post by jvr
Post by Ganzhinterseher
1/1, 1/2, 2/1, 1/3, 3/1, 1/4, 2/3, 3/2, 4/1, 1/5, 5/1, 1/6, ... .
Bisher hat niemand den Grenzwert des Verhältnisses der Anzahl der positiven rationalen Zahlen aus dem ersten und dem n-ten Einheitsintervall, also aus (0, 1] und (n, n+1], ausrechnen können, nicht einmal das Verhältnis aus (0, 1] und (1000, 1001]. Aber das ist sicher größer als 100.
Kann wenigstens jemand die Bedeutung dieses Verhältnisses interpretieren?
Sie halten diese triviale Aufgabe also für schwierig?
Kinder, da ist etwas faul! So ein kleines Hirn und so ein großes Maul!
Seit dem 6. 6. ist eine Woche verflossen. Kommt da noch was aus dem großen Maul?
Gruß, WM
Die Voraussetzung dafür, dass ich Ihnen mit Ihrem Trivialproblem helfe, ist,
dass Sie sowohl hier als auch in den englischsprachigen Gruppen klar und
deutlich erklären, dass Sie zu doof sind es selber zu lösen.

Ausserdem, dass Sie so blöd sind, dass Sie wirklich glauben, die Existenz
dieser Grenzwerte hätte irgendwelche Konsequenzen für die Abzählbarkeit der
rationalen Zahlen.
Ganzhinterseher
2020-06-13 17:13:32 UTC
Permalink
Post by jvr
Post by Ganzhinterseher
Post by jvr
Post by Ganzhinterseher
1/1, 1/2, 2/1, 1/3, 3/1, 1/4, 2/3, 3/2, 4/1, 1/5, 5/1, 1/6, ... .
Bisher hat niemand den Grenzwert des Verhältnisses der Anzahl der positiven rationalen Zahlen aus dem ersten und dem n-ten Einheitsintervall, also aus (0, 1] und (n, n+1], ausrechnen können, nicht einmal das Verhältnis aus (0, 1] und (1000, 1001]. Aber das ist sicher größer als 100.
Kann wenigstens jemand die Bedeutung dieses Verhältnisses interpretieren?
Sie halten diese triviale Aufgabe also für schwierig?
Kinder, da ist etwas faul! So ein kleines Hirn und so ein großes Maul!
Seit dem 6. 6. ist eine Woche verflossen. Kommt da noch was aus dem großen Maul?
Die Voraussetzung dafür, dass ich Ihnen mit Ihrem Trivialproblem helfe, ist,
dass Sie sowohl hier als auch in den englischsprachigen Gruppen klar und
deutlich erklären, dass Sie zu doof sind es selber zu lösen.
Also nur ein großes Maul. Dachte ich's doch.
Post by jvr
Ausserdem, dass Sie so blöd sind, dass Sie wirklich glauben, die Existenz
dieser Grenzwerte hätte irgendwelche Konsequenzen für die Abzählbarkeit der
rationalen Zahlen.
Da diese Abzählbarkeit nur ein närrischer Aberglaube ist, hat der Grenzwert tatsächlich keine Konsequenzen, denn närrischen Aberglauben kann man nicht ausrotten. Mathematisch steht aber für Ungläubige jedenfalls fest, dass das Verhältnis der Grenzwerte genau das Verhältnis der nummerierten Brüche angibt.

Gruß, WM
jvr
2020-06-13 19:56:48 UTC
Permalink
Post by Ganzhinterseher
Post by jvr
Post by Ganzhinterseher
Post by jvr
Post by Ganzhinterseher
1/1, 1/2, 2/1, 1/3, 3/1, 1/4, 2/3, 3/2, 4/1, 1/5, 5/1, 1/6, ... .
Bisher hat niemand den Grenzwert des Verhältnisses der Anzahl der positiven rationalen Zahlen aus dem ersten und dem n-ten Einheitsintervall, also aus (0, 1] und (n, n+1], ausrechnen können, nicht einmal das Verhältnis aus (0, 1] und (1000, 1001]. Aber das ist sicher größer als 100.
Kann wenigstens jemand die Bedeutung dieses Verhältnisses interpretieren?
Sie halten diese triviale Aufgabe also für schwierig?
Kinder, da ist etwas faul! So ein kleines Hirn und so ein großes Maul!
Seit dem 6. 6. ist eine Woche verflossen. Kommt da noch was aus dem großen Maul?
Die Voraussetzung dafür, dass ich Ihnen mit Ihrem Trivialproblem helfe, ist,
dass Sie sowohl hier als auch in den englischsprachigen Gruppen klar und
deutlich erklären, dass Sie zu doof sind es selber zu lösen.
Also nur ein großes Maul. Dachte ich's doch.
Post by jvr
Ausserdem, dass Sie so blöd sind, dass Sie wirklich glauben, die Existenz
dieser Grenzwerte hätte irgendwelche Konsequenzen für die Abzählbarkeit der
rationalen Zahlen.
Da diese Abzählbarkeit nur ein närrischer Aberglaube ist, hat der Grenzwert tatsächlich keine Konsequenzen, denn närrischen Aberglauben kann man nicht ausrotten. Mathematisch steht aber für Ungläubige jedenfalls fest, dass das Verhältnis der Grenzwerte genau das Verhältnis der nummerierten Brüche angibt.
Gruß, WM
Also gut, Herr Professor Doktor Ganzhinterwalden, wir werden schrittweise vorgehen.

Sie haben also Mühe zu bestimmen für wieviele Werte k, bei gegebenem n und m, die Ungleichung m < (n-k)/k < m + 1 gilt? Und das ist Ihnen nicht
einmal peinlich. Habe ich das richtig verstanden?
Alfred Flaßhaar
2020-06-06 12:12:46 UTC
Permalink
Post by Ganzhinterseher
1/1, 1/2, 2/1, 1/3, 3/1, 1/4, 2/3, 3/2, 4/1, 1/5, 5/1, 1/6, ... .
Bisher hat niemand den Grenzwert des Verhältnisses der Anzahl der positiven rationalen Zahlen aus dem ersten und dem n-ten Einheitsintervall, also aus (0, 1] und (n, n+1], ausrechnen können, nicht einmal das Verhältnis aus (0, 1] und (1000, 1001]. Aber das ist sicher größer als 100.
Warum verwendest Du den Begriff "Grenzwert"? Möchtest Du ausgehend von
den "Verhältnissen" endlicher Zahlenmengen aus den betreffenden
Intervallen das asymptotische Verhalten dieses Verhältnisses bestimmen?
Post by Ganzhinterseher
Kann wenigstens jemand die Bedeutung dieses Verhältnisses interpretieren?
Bitte vertausche die Wörter "wenigstens" und "jemand" in der
Reihenfolge. Denn Dir geht es ja wohl um das "Verhältnis" und nicht
darum, ob "jemand" existiert.

Gruß, Alfred Flaßhaar
jvr
2020-06-06 13:12:27 UTC
Permalink
Post by Alfred Flaßhaar
Post by Ganzhinterseher
1/1, 1/2, 2/1, 1/3, 3/1, 1/4, 2/3, 3/2, 4/1, 1/5, 5/1, 1/6, ... .
Bisher hat niemand den Grenzwert des Verhältnisses der Anzahl der positiven rationalen Zahlen aus dem ersten und dem n-ten Einheitsintervall, also aus (0, 1] und (n, n+1], ausrechnen können, nicht einmal das Verhältnis aus (0, 1] und (1000, 1001]. Aber das ist sicher größer als 100.
Warum verwendest Du den Begriff "Grenzwert"? Möchtest Du ausgehend von
den "Verhältnissen" endlicher Zahlenmengen aus den betreffenden
Intervallen das asymptotische Verhalten dieses Verhältnisses bestimmen?
Post by Ganzhinterseher
Kann wenigstens jemand die Bedeutung dieses Verhältnisses interpretieren?
Bitte vertausche die Wörter "wenigstens" und "jemand" in der
Reihenfolge. Denn Dir geht es ja wohl um das "Verhältnis" und nicht
darum, ob "jemand" existiert.
Gruß, Alfred Flaßhaar
Das geht entschieden zu weit. Erst verlangt man von ihm, dass er kapiert, was
er schreibt; dann, dass er den englischen Konjunktiv beherrscht; und jetzt
noch die deutsche Syntax.
Das geht wirklich zu weit. Und alles nur weil er zu doof ist diese triviale
Aufgabe zu lösen, die aber mit dem Grundübel garnichts zu tun hat.
Ganzhinterseher
2020-06-07 14:58:16 UTC
Permalink
Post by Alfred Flaßhaar
Post by Ganzhinterseher
1/1, 1/2, 2/1, 1/3, 3/1, 1/4, 2/3, 3/2, 4/1, 1/5, 5/1, 1/6, ... .
Bisher hat niemand den Grenzwert des Verhältnisses der Anzahl der positiven rationalen Zahlen aus dem ersten und dem n-ten Einheitsintervall, also aus (0, 1] und (n, n+1], ausrechnen können, nicht einmal das Verhältnis aus (0, 1] und (1000, 1001]. Aber das ist sicher größer als 100.
Warum verwendest Du den Begriff "Grenzwert"?
Weil ich das Verhältnis für alle Indizes wissen möchte.
Post by Alfred Flaßhaar
Möchtest Du ausgehend von
den "Verhältnissen" endlicher Zahlenmengen aus den betreffenden
Intervallen das asymptotische Verhalten dieses Verhältnisses bestimmen?
Mathematisch ist das der übliche Weg. Für die Funktion f(x) = x/sinx bestimmen wir den Grenzwert ausgehend von den Verhältnissen endlicher x für x --> 0.
Post by Alfred Flaßhaar
Bitte vertausche die Wörter "wenigstens" und "jemand" in der
Reihenfolge. Denn Dir geht es ja wohl um das "Verhältnis" und nicht
darum, ob "jemand" existiert.
Eine völlig ausreichenden Abschätzung für den Grenzwert ist mir bekannt. Mir geht es um die Frage, ob wenigstens einer unter Euch genug Verstand besitzt, um die Folgerung aus dem Offensichtlichen zu ziehen.

Gruß, WM
Mostowski Collapse
2020-06-06 12:42:25 UTC
Permalink
Sie müssen nicht rational Zahlen hinzuziehen, um mit
Verhältnissen zu spielen. Betrachten Sie:

(An) = (1, 4, 7, 10, 13, ...)

(Bn) = (2, 3, 5, 6, 8, 9, 11, 12, ...)

Wenn wir die Elemente zählen, die unterhalb einer Grenze sind:

(#An) = (1, 1, 1, 2, 2, 2, 3, 3, 3, ...)

(#Bn) = (0, 1, 2, 2, 3, 4, 4, 5, 6, ...)

Dann:

lim n->oo An/Bn = +oo

lim n->oo #An/#Bn = 0

Das ist doch Paradox, dass einmal mehr weniger ist!
Post by Ganzhinterseher
1/1, 1/2, 2/1, 1/3, 3/1, 1/4, 2/3, 3/2, 4/1, 1/5, 5/1, 1/6, ... .
Bisher hat niemand den Grenzwert des Verhältnisses der Anzahl der positiven rationalen Zahlen aus dem ersten und dem n-ten Einheitsintervall, also aus (0, 1] und (n, n+1], ausrechnen können, nicht einmal das Verhältnis aus (0, 1] und (1000, 1001]. Aber das ist sicher größer als 100.
Kann wenigstens jemand die Bedeutung dieses Verhältnisses interpretieren?
Gruß, WM
Mostowski Collapse
2020-06-06 13:02:20 UTC
Permalink
Corr.:

lim n->oo An/Bn = 2

lim n->oo #An/#Bn = 1/2

An=n*3-2, Bn=n+1+floor((n-1)/2).

#An=1+floor((n-1)/3), #Bn=floor(2*n/3).
Post by Mostowski Collapse
Sie müssen nicht rational Zahlen hinzuziehen, um mit
(An) = (1, 4, 7, 10, 13, ...)
(Bn) = (2, 3, 5, 6, 8, 9, 11, 12, ...)
(#An) = (1, 1, 1, 2, 2, 2, 3, 3, 3, ...)
(#Bn) = (0, 1, 2, 2, 3, 4, 4, 5, 6, ...)
lim n->oo An/Bn = +oo
lim n->oo #An/#Bn = 0
Das ist doch Paradox, dass einmal mehr weniger ist!
Post by Ganzhinterseher
1/1, 1/2, 2/1, 1/3, 3/1, 1/4, 2/3, 3/2, 4/1, 1/5, 5/1, 1/6, ... .
Bisher hat niemand den Grenzwert des Verhältnisses der Anzahl der positiven rationalen Zahlen aus dem ersten und dem n-ten Einheitsintervall, also aus (0, 1] und (n, n+1], ausrechnen können, nicht einmal das Verhältnis aus (0, 1] und (1000, 1001]. Aber das ist sicher größer als 100.
Kann wenigstens jemand die Bedeutung dieses Verhältnisses interpretieren?
Gruß, WM
Mostowski Collapse
2020-06-06 13:07:49 UTC
Permalink
Zu allem Überfluss:

{An} ∩ {Bn} = {}

{An} ∪ {Bn} = N

Es handelt sich um eine vollständige Partition
von N, und je nachdem wie wir Messen, ist die
eine Partition doppelt so gross wie die andere

Partition oder halb so gross wie die andere
Partition. Nach Augsburg Crank Institut deutet
das sehr schwer darauf hin dass die Mainstream

Mathematik inkonsistent ist, und nicht das
Prof Muckefunk ne Mattscheibe hat.
Post by Mostowski Collapse
lim n->oo An/Bn = 2
lim n->oo #An/#Bn = 1/2
An=n*3-2, Bn=n+1+floor((n-1)/2).
#An=1+floor((n-1)/3), #Bn=floor(2*n/3).
Post by Mostowski Collapse
Sie müssen nicht rational Zahlen hinzuziehen, um mit
(An) = (1, 4, 7, 10, 13, ...)
(Bn) = (2, 3, 5, 6, 8, 9, 11, 12, ...)
(#An) = (1, 1, 1, 2, 2, 2, 3, 3, 3, ...)
(#Bn) = (0, 1, 2, 2, 3, 4, 4, 5, 6, ...)
lim n->oo An/Bn = +oo
lim n->oo #An/#Bn = 0
Das ist doch Paradox, dass einmal mehr weniger ist!
Post by Ganzhinterseher
1/1, 1/2, 2/1, 1/3, 3/1, 1/4, 2/3, 3/2, 4/1, 1/5, 5/1, 1/6, ... .
Bisher hat niemand den Grenzwert des Verhältnisses der Anzahl der positiven rationalen Zahlen aus dem ersten und dem n-ten Einheitsintervall, also aus (0, 1] und (n, n+1], ausrechnen können, nicht einmal das Verhältnis aus (0, 1] und (1000, 1001]. Aber das ist sicher größer als 100.
Kann wenigstens jemand die Bedeutung dieses Verhältnisses interpretieren?
Gruß, WM
Ganzhinterseher
2020-06-07 16:03:20 UTC
Permalink
Post by Mostowski Collapse
{An} ∩ {Bn} = {}
{An} ∪ {Bn} = N
Es handelt sich um eine vollständige Partition
von N, und je nachdem wie wir Messen, ist die
eine Partition doppelt so gross wie die andere
Partition oder halb so gross wie die andere
Partition.
Hier gibt es nur eine einzige Lösung:

lim k --> oo [
|{x ∈ ℝ | 0 < x =< 1} ∩ {q_1, q_2, ..., q_k}|
/
|{x ∈ ℝ | 1000 < x =< 1001} ∩ {q_1, q_2, ..., q_k}|]

wobei die q_i der Cantorschen Folge

1/1, 1/2, 2/1, 1/3, 3/1, 1/4, 2/3, 3/2, 4/1, 1/5, 5/1, 1/6, ...

entnommen sind.

Das Problem ist also von Deinem verschieden.

Gruß, WM
Mostowski Collapse
2020-06-07 16:10:27 UTC
Permalink
Die Verwirrung ist die gleiche. Der vergebliche
Versuch unedliche Mengen eindeutig zu messen,

basierend auf irgendwelchen ad-hoc Prozessen.
Garbage in, Garbage out.
Post by Ganzhinterseher
Post by Mostowski Collapse
{An} ∩ {Bn} = {}
{An} ∪ {Bn} = N
Es handelt sich um eine vollständige Partition
von N, und je nachdem wie wir Messen, ist die
eine Partition doppelt so gross wie die andere
Partition oder halb so gross wie die andere
Partition.
lim k --> oo [
|{x ∈ ℝ | 0 < x =< 1} ∩ {q_1, q_2, ..., q_k}|
/
|{x ∈ ℝ | 1000 < x =< 1001} ∩ {q_1, q_2, ..., q_k}|]
wobei die q_i der Cantorschen Folge
1/1, 1/2, 2/1, 1/3, 3/1, 1/4, 2/3, 3/2, 4/1, 1/5, 5/1, 1/6, ...
entnommen sind.
Das Problem ist also von Deinem verschieden.
Gruß, WM
Mostowski Collapse
2020-06-07 16:13:43 UTC
Permalink
Wenn eine Bijektion zwischen Q und N aufgestellt
wird, besteht nirgends ein Anspruch dass dies

irgendwelche Messungen hervorruft. Die einzige
Message die eine Bijektion veranlassen kann, ist

Cardinality. Wir können zeigen dass |Q|=|N|=א0
Aber andere Messungen sind ja nicht impliziert.
Post by Mostowski Collapse
Die Verwirrung ist die gleiche. Der vergebliche
Versuch unedliche Mengen eindeutig zu messen,
basierend auf irgendwelchen ad-hoc Prozessen.
Garbage in, Garbage out.
Post by Ganzhinterseher
Post by Mostowski Collapse
{An} ∩ {Bn} = {}
{An} ∪ {Bn} = N
Es handelt sich um eine vollständige Partition
von N, und je nachdem wie wir Messen, ist die
eine Partition doppelt so gross wie die andere
Partition oder halb so gross wie die andere
Partition.
lim k --> oo [
|{x ∈ ℝ | 0 < x =< 1} ∩ {q_1, q_2, ..., q_k}|
/
|{x ∈ ℝ | 1000 < x =< 1001} ∩ {q_1, q_2, ..., q_k}|]
wobei die q_i der Cantorschen Folge
1/1, 1/2, 2/1, 1/3, 3/1, 1/4, 2/3, 3/2, 4/1, 1/5, 5/1, 1/6, ...
entnommen sind.
Das Problem ist also von Deinem verschieden.
Gruß, WM
Ganzhinterseher
2020-06-07 18:49:51 UTC
Permalink
Post by Mostowski Collapse
Wenn eine Bijektion zwischen Q und N aufgestellt
wird, besteht nirgends ein Anspruch dass dies
irgendwelche Messungen hervorruft.
Es sollte aber ein Anspruch auf verständige Behandlung des Unendlichen bestehen. Die meisten Zahlen in einer aktual unendlichen Menge sind nicht definierbar. Das erkennt man am einfachsten an der Tatsache, dass für alle definierbaren Zahlen gilt

∀k ∈ ℕ_def: |ℕ \ {1, 2, 3, ..., k}| = ℵo.

Ist das wirklich so schwer zu begreifen? Man kann auch alle definierbaren Zahlen in einer potentiell unendlichen Kollektion ℕ_def zusammenfassen und dann schreiben

|ℕ \ ℕ_def| = ℵo.

Gruß, WM
Juergen Ilse
2020-06-07 19:08:32 UTC
Permalink
Hallo,
Post by Ganzhinterseher
Post by Mostowski Collapse
Wenn eine Bijektion zwischen Q und N aufgestellt
wird, besteht nirgends ein Anspruch dass dies
irgendwelche Messungen hervorruft.
Es sollte aber ein Anspruch auf verständige Behandlung des Unendlichen
bestehen.
Was fuer ein Schwachsinn soll damit jetzt wieder gemeint sein?
Post by Ganzhinterseher
Die meisten Zahlen in einer aktual unendlichen Menge sind nicht definierbar.
IHRE daemliche Wahnvorstellung von "nicht definierbren Zahlen" ist einfach
nur inkonsistenter Unfug. SIE schaffen es janoch nicht einmal, *eine* kon-
sistente definition fuer "undefinierbare natuerliche Zahlen" anzugeben.
Statt dessen lifern SIE etliche verschiedene, von denen SIOE noch nicht
einmal nachweisen, dass sie aequivalent sind. SIE mathematischer vollpfosten
haben noch nicht einmal die vollstaendige Induktion verstanden, sonst wuerden
SIE ja begreifen, dass sich mittels vollstaendiger Induktion beweisen laesst,
dass *alle* natuerlichen Zahlen eine Dezimaldarstellung besitzen (auch wenn
man sie moeglicherweise nicht immer aufschreiben kann, weil man da ein
"Resourcenproblem" haette (man fuer manche Zahlen nicht genug Papier, Tinte,
Lebenszeit, ... zur Verfuegung haette).
Post by Ganzhinterseher
Das erkennt man am einfachsten an der Tatsache, dass für alle definierbaren Zahlen gilt
∀k ∈ ℕ_def: |ℕ \ {1, 2, 3, ..., k}| = ℵo.
SIE mathematischer Vollidiot sind sogar zu bloed, um den Unterschied zwischen
endlich und unendlich zu begreifen, sonst wuerden SIE nicht solchen Schwach-
sinn schreiben.
Post by Ganzhinterseher
Ist das wirklich so schwer zu begreifen?
Daran gibt es nichts zu begreifen, weil SIE ihren daemlichen Wahn von
!undefinierbren Zahlen" niemals in eine wirklich konsistente Theorie
hineinbekommen. Und SIE sind zu beschrankt, um IHRE eigene Unfaehigkeit
zu begreifeen. Dazu passt eigentlich nur noch das schoene Gedicht von
Wilhelm Busch:

Wenn einer, der mit Muehe kaum
geklettert ist auf einen Baum
schon meint, dass er ein Vogel waer'
so irrt sich der.

Wobei SIE bzgl. Mathematik noch nicht einmal das "klettern auf einen Baum"
geschafft haben.

Tschuess,
Juergen Ilse ***@usenet-verwaltung.de)
WM
2020-06-07 20:11:26 UTC
Permalink
Post by Juergen Ilse
Hallo,
Post by Ganzhinterseher
Post by Mostowski Collapse
Wenn eine Bijektion zwischen Q und N aufgestellt
wird, besteht nirgends ein Anspruch dass dies
irgendwelche Messungen hervorruft.
Es sollte aber ein Anspruch auf verständige Behandlung des Unendlichen
bestehen.
Die meisten Zahlen in einer aktual unendlichen Menge sind nicht definierbar.
Das erkennt man am einfachsten an der Tatsache, dass für alle definierbaren Zahlen gilt
∀k ∈ ℕ_def: |ℕ \ {1, 2, 3, ..., k}| = ℵo.
Unterschied zwischen
endlich und unendlich
Ist leicht erkennbar, wenn man alle definierbaren Zahlen k ∈ ℕ_def zusammenfasst

ℕ \ ℕ_def = ℵo.

Oder weißt Du eine definierbare Zahl, die dort fehlen würde? Bitte liefere keine weiteren Beweise für Deine mangelhafte Erziehung, sondern ein Beispiel einer definierbaren Zahl, die hier fehl am Platze wäre:

ℕ \ ℕ_def = ℵo.

Gruß, WM
Ganzhinterseher
2020-06-07 18:42:30 UTC
Permalink
Post by Mostowski Collapse
Die Verwirrung ist die gleiche.
Von der Mathematik, insbesondere von Grenzprozessen und Grenzwerten, haben sich schon viele verwirren lassen. Das musst Du wohl einfach so hinnehmen.
Post by Mostowski Collapse
Der vergebliche
Versuch unendliche Mengen eindeutig zu messen,
wurde von Cantor angestellt, wie man sofort erkennt, wenn man mathematische Methoden anwendet. Merkwürdig nur, dass sich so viele von der Behauptung haben bluffen lassen, die Unendlichkeit sei vollendbar.

Gruß, WM
h***@gmail.com
2020-06-08 15:11:20 UTC
Permalink
Post by Ganzhinterseher
Von der Mathematik, insbesondere von Grenzprozessen und Grenzwerten, haben sich schon viele verwirren lassen.
Ganz voran natürlich unser Herr Professor Mückenheim, der meint, aus irgendeinem asymptotischen Verhalten von Kardinalitäten einen Schluss auf das asymptotische Verhalten der zugrundegelegten Mengen ableiten zu können.
jvr
2020-06-06 13:14:01 UTC
Permalink
Post by Mostowski Collapse
Sie müssen nicht rational Zahlen hinzuziehen, um mit
(An) = (1, 4, 7, 10, 13, ...)
(Bn) = (2, 3, 5, 6, 8, 9, 11, 12, ...)
(#An) = (1, 1, 1, 2, 2, 2, 3, 3, 3, ...)
(#Bn) = (0, 1, 2, 2, 3, 4, 4, 5, 6, ...)
lim n->oo An/Bn = +oo
lim n->oo #An/#Bn = 0
Das ist doch Paradox, dass einmal mehr weniger ist!
Post by Ganzhinterseher
1/1, 1/2, 2/1, 1/3, 3/1, 1/4, 2/3, 3/2, 4/1, 1/5, 5/1, 1/6, ... .
Bisher hat niemand den Grenzwert des Verhältnisses der Anzahl der positiven rationalen Zahlen aus dem ersten und dem n-ten Einheitsintervall, also aus (0, 1] und (n, n+1], ausrechnen können, nicht einmal das Verhältnis aus (0, 1] und (1000, 1001]. Aber das ist sicher größer als 100.
Kann wenigstens jemand die Bedeutung dieses Verhältnisses interpretieren?
Gruß, WM
Wieso paradox? Weniger ist mehr. Das weiß doch jedes Kind.
Ganzhinterseher
2020-06-07 15:51:41 UTC
Permalink
Post by Mostowski Collapse
Sie müssen nicht rational Zahlen hinzuziehen, um mit
Verhältnissen zu spielen.
Die Frage ist ganz konkret auf die rationalen Zahlen bezogen, die sich in der Abzählung (angeblich aller) positiven rationalen Zahlen befinden: Was ist die Bedeutung der unterschiedlichen Trefferzahl?

Gruß, WM
Me
2020-06-06 12:54:05 UTC
Permalink
Post by Ganzhinterseher
Bisher hat niemand den Grenzwert des Verhältnisses der Anzahl der positiven
rationalen Zahlen aus dem ersten und dem n-ten Einheitsintervall, also aus
(0, 1] und (n, n+1], ausrechnen können [...]
Das liegt womöglich daran, dass unklar ist, was Du mit dem "Grenzwert des Verhältnisses der Anzahl der positiven rationalen Zahlen aus dem ersten und dem n-ten Einheitsintervall" überhaupt meinst. Kann man das auch in "mathematischer Sprache" ausdrücken?

Hinweis: "About 90% of the effort in conversing with you is coming up with things you might be trying to say."

(Gefunden in sci.math in einer Antwort auf ein Post von Ganzhinterseher)
Ganzhinterseher
2020-06-07 16:01:13 UTC
Permalink
Post by Me
Post by Ganzhinterseher
Bisher hat niemand den Grenzwert des Verhältnisses der Anzahl der positiven
rationalen Zahlen aus dem ersten und dem n-ten Einheitsintervall, also aus
(0, 1] und (n, n+1], ausrechnen können [...]
Das liegt womöglich daran, dass unklar ist, was Du mit dem "Grenzwert des Verhältnisses der Anzahl der positiven rationalen Zahlen aus dem ersten und dem n-ten Einheitsintervall" überhaupt meinst. Kann man das auch in "mathematischer Sprache" ausdrücken?
Dafür gibt es das Wort "Grenzwert", dessen Bedeutung ganz klar definiert ist. Gesucht ist

lim k --> oo [
|{x ∈ ℝ | 0 < x =< 1} ∩ {q_1, q_2, ..., q_k}|
/
|{x ∈ ℝ | 1000 < x =< 1001} ∩ {q_1, q_2, ..., q_k}|]

wobei die q_i der Cantorschen Folge

1/1, 1/2, 2/1, 1/3, 3/1, 1/4, 2/3, 3/2, 4/1, 1/5, 5/1, 1/6, ...

entnommen sind.

Gruß, WM
Juergen Ilse
2020-06-07 19:18:13 UTC
Permalink
Hallo,
Post by Ganzhinterseher
Dafür gibt es das Wort "Grenzwert", dessen Bedeutung ganz klar definiert ist.
Dann nennen SIE doch vielleicht zur Abwechselung mal eine mathematisch
korrekte Definition, z.B. eine matheatisch korrekte Definition des Begriffs
"Grenzwert".

Tschuess,
Juergen ilse ***@usenet-verwaltung.de)
Ganzhinterseher
2020-06-08 13:48:56 UTC
Permalink
Post by Juergen Ilse
Hallo,
Post by Ganzhinterseher
Dafür gibt es das Wort "Grenzwert", dessen Bedeutung ganz klar definiert ist.
Dann nennen SIE doch vielleicht zur Abwechselung mal eine mathematisch
korrekte Definition, z.B. eine matheatisch korrekte Definition des Begriffs
"Grenzwert".
Der Grenzwert oder Limes einer Folge von Zahlen ist eine Zahl, der die Folge beliebig nah kommt. (Wikipedia)

Im vorliegenden Falle geht es um eine Folge von Quotienten:

lim k --> oo [
|{x ∈ ℝ | 0 < x =< 1} ∩ {q_1, q_2, ..., q_k}|
/
|{x ∈ ℝ | 1000 < x =< 1001} ∩ {q_1, q_2, ..., q_k}|]

wobei die q_i der Cantorschen Folge

1/1, 1/2, 2/1, 1/3, 3/1, 1/4, 2/3, 3/2, 4/1, 1/5, 5/1, 1/6, ...

entnommen sind.

Ist das für Dich verständlich hingeschrieben?

Gruß, WM
Juergen Ilse
2020-06-08 14:09:13 UTC
Permalink
Hallo,
Post by Ganzhinterseher
Post by Juergen Ilse
Post by Ganzhinterseher
Dafür gibt es das Wort "Grenzwert", dessen Bedeutung ganz klar definiert ist.
Dann nennen SIE doch vielleicht zur Abwechselung mal eine mathematisch
korrekte Definition, z.B. eine matheatisch korrekte Definition des Begriffs
"Grenzwert".
Der Grenzwert oder Limes einer Folge von Zahlen ist eine Zahl, der die Folge beliebig nah kommt. (Wikipedia)
Das ist eine sehr "umgangssprachliche" und keine mathematisch exakte
Definition. Wenn bspw. ein Wert von der Folge erreicht wird, ist die Folge
diesem Wert ja "beliebig nahe" gekommen. In der Formulierung ist aber z.B.
nicht beruecksichtigt, dass sich nachfolgende Folgenglieder auch wieder von
diesem Wert wegbewegen koennten, und das waere dieser Wert womoeglich kein
Grenzwert mehr ... Deswegen fragte ich nach einer mathematischen Definition
des Begriffs (eine formale Definition, die keine solche "Fehlinterpretationen"
mehr zulaesst ...).
Post by Ganzhinterseher
lim k --> oo [
|{x ∈ ℝ | 0 < x =< 1} ∩ {q_1, q_2, ..., q_k}|
/
|{x ∈ ℝ | 1000 < x =< 1001} ∩ {q_1, q_2, ..., q_k}|]
wobei die q_i der Cantorschen Folge
1/1, 1/2, 2/1, 1/3, 3/1, 1/4, 2/3, 3/2, 4/1, 1/5, 5/1, 1/6, ...
entnommen sind.
Wenn man einen Grenzwert bestimmen will, muss man zuerst einmal nachweisen,
dass dieser Grenzwert ueberhaupt existiert oder es sich moeglicherweise um
eine divergente Folge handelt. Wo ist IHR Beweis der Existenz eines Grenz-
wertes )weil ohne einen solchen Nachweis die Frage nac hdem Grenzwert selbst
voellig unsinnig waere).

Tschuess,
Juergen Ilse (***@usenet-verwaltung.de)
Juergen Ilse
2020-06-08 15:32:19 UTC
Permalink
Hallo,
Post by Juergen Ilse
Post by Ganzhinterseher
Post by Juergen Ilse
Post by Ganzhinterseher
Dafür gibt es das Wort "Grenzwert", dessen Bedeutung ganz klar definiert ist.
Dann nennen SIE doch vielleicht zur Abwechselung mal eine mathematisch
korrekte Definition, z.B. eine matheatisch korrekte Definition des Begriffs
"Grenzwert".
Der Grenzwert oder Limes einer Folge von Zahlen ist eine Zahl, der die Folge beliebig nah kommt. (Wikipedia)
Das ist eine sehr "umgangssprachliche" und keine mathematisch exakte
Definition. Wenn bspw. ein Wert von der Folge erreicht wird, ist die Folge
diesem Wert ja "beliebig nahe" gekommen. In der Formulierung ist aber z.B.
nicht beruecksichtigt, dass sich nachfolgende Folgenglieder auch wieder von
diesem Wert wegbewegen koennten, und das waere dieser Wert womoeglich kein
Grenzwert mehr ... Deswegen fragte ich nach einer mathematischen Definition
des Begriffs (eine formale Definition, die keine solche "Fehlinterpretationen"
mehr zulaesst ...).
Post by Ganzhinterseher
lim k --> oo [
|{x ∈ ℝ | 0 < x =< 1} ∩ {q_1, q_2, ..., q_k}|
/
|{x ∈ ℝ | 1000 < x =< 1001} ∩ {q_1, q_2, ..., q_k}|]
wobei die q_i der Cantorschen Folge
1/1, 1/2, 2/1, 1/3, 3/1, 1/4, 2/3, 3/2, 4/1, 1/5, 5/1, 1/6, ...
entnommen sind.
Wenn man einen Grenzwert bestimmen will, muss man zuerst einmal nachweisen,
dass dieser Grenzwert ueberhaupt existiert oder es sich moeglicherweise um
eine divergente Folge handelt. Wo ist IHR Beweis der Existenz eines Grenz-
wertes) weil ohne einen solchen Nachweis die Frage nach dem Grenzwert selbst
voellig unsinnig waere).
Ach ja, und bevor ich es vergesse: selbst wenn dieser Grenzwert existiert,
besagt er (unabhaengig von seinem Wert) *nichts* darueber aus, ob die Can-
torsche Abbildung bisjektiv ist oder nicht. Auch wenn SIE jetzt wieder an-
fangen werden, irgendetwas mit angeblicher "Logik" daherzufasseln, um doch
einen Zusammenhang herzustellen: Nein, einen solchen Zusammenhang gibt es
*nicht*. Das die Abbildung bijektiv ist, ist aber leicht zu beweisen.

Tschuess,
Juergen Ilse (***@usenet-verwaltung.de)
PS: Wenn der Grenzwert existieren sollte, liegt er mit Sicherheit >=1,
denn egal, wie weit man geht: Die Diagonale und alles oeberhalb der Diagonale
liegt mit Sicherheit *immer* im halboffenen Intervall ]0;1]. Egal, wie weit
man die Diagonalen durchzaehlt, ist also *immer* ungefaehr die Haelfte der
bis dahin abgezaehlten Elemente im halboffenen Intervall ]0;1] und damit mit
Sicherheit nicht im halboffenen Intervall ]1000;1001]. Darueber, ob die
Abbildung bijektiv ist oder nicht, laesst diese Betrachtung allerdings kei-
nerlei Aussage zu.
Juergen Ilse
2020-06-08 16:17:52 UTC
Permalink
Hallo,
Post by Juergen Ilse
Ach ja, und bevor ich es vergesse: selbst wenn dieser Grenzwert existiert,
besagt er (unabhaengig von seinem Wert) *nichts* darueber aus, ob die Can-
torsche Abbildung bisjektiv ist oder nicht. Auch wenn SIE jetzt wieder an-
fangen werden, irgendetwas mit angeblicher "Logik" daherzufasseln, um doch
einen Zusammenhang herzustellen: Nein, einen solchen Zusammenhang gibt es
*nicht*. Das die Abbildung bijektiv ist, ist aber leicht zu beweisen.
PS: Wenn der Grenzwert existieren sollte, liegt er mit Sicherheit >=1,
denn egal, wie weit man geht: Die Diagonale und alles oeberhalb der Diagonale
liegt mit Sicherheit *immer* im halboffenen Intervall ]0;1]. Egal, wie weit
man die Diagonalen durchzaehlt, ist also *immer* ungefaehr die Haelfte der
bis dahin abgezaehlten Elemente im halboffenen Intervall ]0;1] und damit mit
Sicherheit nicht im halboffenen Intervall ]1000;1001]. Darueber, ob die
Abbildung bijektiv ist oder nicht, laesst diese Betrachtung allerdings kei-
nerlei Aussage zu.
Der Grenzwert existiert nicht (als "eigentlicher Grenzwert" bzw. als "unei-
gentlicher Grenzwert" waere er "unendlich".
Beweis:
Den ersten Quotienten aus dem Intervall findet man erst bei q_5050, und auch
in den darauf folgenden 1000 Diagonalen findet man nur jeweils ein Element
aus diesem Intervall auf der jeweiligen Diagonale. Damit steigt die Anzahl
der Elemente aus dem Intervall ]0;1] auf einer Diagonale linear mit der Nummer
der jeweiligen Diagonale (bei jeder zweiten Diagonale nimmt dieser Wert um 1
zu), bei den Elementen aus dem Intervall ]1000;1001] nimmt die Anzahl der
Elemente aus diesem Intervall maximal alle 1000 Diagonalen um 1 zu (und in
der tausendsten Diagonale, die mit dem Element q_5050 beginnen wuerde waere
erstmalig ein Element aus dem Intervall ]1000;1001] enthalten). Die Anzahl
der Elemente aus dem Intervall ]0;1] steigen mit steigender Zahl der "durch-
numerierten Diagonalen" also sehr viel staerker an, als die Anzahl der
Elemente aus dem Intervall ]1000;1001].

Nur hat das nicht den geringsten Einfluss auf die Bijektivitaet der Abbildung
von der Menge der natuerlichen Zahlen auf die Menge der rationalen Zahlen,
warum sollte da auch ein Zusammenhang existieren?

Numeriert man im Gegensatz zu Cantor alle Diagonalen von links unten nach
rechts oben durch, kann man den Index ausrechnen, den der Bruch p/q in der
Abzaehlung hat:

Er stuende in der "p-ten Zeile" und "q-ten Spalte", damit also in der
"(p+q-1)-ten" Diagonale. Auf dieser Diagonale wueder er dann den !q-ten"
Platz belegen. Da die Summe aller Brueche in den ersten (p+q-2) Diagonalen
gleich (p+q-2)*(p+q-3)/2 ist (Summenformel fuer arichtmetische Reihe).
Damit ergibt sich der Index des Bruches p/q zu q+(p+q-2)*(p+q-3)/2. Das
ist mit absoluter Sicherheit eine natuerliche Zahl. Ich kann also zu jedem
positivn Bruch bei *dieser* Abzaehlung genau den Index des Bruches bai der
Abzaehlung berechnen. Damit haben wir jetzt also sogar eine geschlossene
Formel fuer die bisjektive Abbildung der positiven Brueche auf die natuer-
lichen Zahlen:

p/q -> q+(p+q-2)*(p+q-3)/2

Dass diese Abbildung bijektiv ist, ergibt aich aus der Konstruktion der
Abbildung (es wird ja stets von links unten nach rechts oben an den Dia-
gonalen der Cantirschen Anordnung der Brueche "entlang gezaehlt").

Tschuess,
Juergen Ilse (***@usenet-verwaltung.de)
PS: Das ist noch keine bijektive Abbildung von den positiven rationalen
Zahlen auf die natuerlichen Zahlen, da wir ja rationale Zahlen mehrfach
zaehlen. Eine bijektive Abbildung wuerde sich nur dann ergeben, wenn man
die "nicht gekuerzten Brueche" auslassen wuerde, aber dann wuerdew unsere
schoene oben stehende geschlossene Formel fuer die Abbildung nicht mehr
zutreffen ...
WM
2020-06-09 15:59:22 UTC
Permalink
Post by Juergen Ilse
Der Grenzwert existiert nicht (als "eigentlicher Grenzwert" bzw. als "unei-
gentlicher Grenzwert" waere er "unendlich".
Das ist nicht ausgeschlossen. Wir sollten einfach den Kehrwert wählen. Der ist ein eigentlicher Grenzwert. Aber 0 hätte ich doch nicht vermutet, sondern doch einen wenn auch sehr kleinen endlichen Wert erwartet.
Post by Juergen Ilse
Nur hat das nicht den geringsten Einfluss auf die Bijektivitaet der Abbildung
von der Menge der natuerlichen Zahlen auf die Menge der rationalen Zahlen,
warum sollte da auch ein Zusammenhang existieren?
Wer die Mengenlehre zu eingehend studiert hat, kann diese Frage wohl nicht umfassend beantworten. Was wäre denn Deiner Vermutung nach eine zutreffende Interpretation des Verhältnisses?

Gruß, WM
Ganzhinterseher
2020-06-13 12:38:47 UTC
Permalink
Post by Juergen Ilse
Der Grenzwert existiert nicht (als "eigentlicher Grenzwert" bzw. als "unei-
gentlicher Grenzwert" waere er "unendlich".
Den ersten Quotienten aus dem Intervall findet man erst bei q_5050, und auch
in den darauf folgenden 1000 Diagonalen findet man nur jeweils ein Element
aus diesem Intervall auf der jeweiligen Diagonale. Damit steigt die Anzahl
der Elemente aus dem Intervall ]0;1] auf einer Diagonale linear mit der Nummer
der jeweiligen Diagonale (bei jeder zweiten Diagonale nimmt dieser Wert um 1
zu), bei den Elementen aus dem Intervall ]1000;1001] nimmt die Anzahl der
Elemente aus diesem Intervall maximal alle 1000 Diagonalen um 1 zu (und in
der tausendsten Diagonale, die mit dem Element q_5050 beginnen wuerde waere
erstmalig ein Element aus dem Intervall ]1000;1001] enthalten). Die Anzahl
der Elemente aus dem Intervall ]0;1] steigen mit steigender Zahl der "durch-
numerierten Diagonalen" also sehr viel staerker an, als die Anzahl der
Elemente aus dem Intervall ]1000;1001].
Ich glaube das nicht, bin aber auch nicht ganz sicher. Mein Argument ist dies:

Die Hälfte aller Brüche stammt aus dem ersten Intervall. Bezeichnen wir sie mit #(0, 1). Also muss die andere Hälfte aus den anderen Intervallen stammen. Also kann nicht jeder Grenzwert
lim #(n, n+1) / #(0, 1) = 0
sein. Stetigkeitsbetrachtungen führen mich dazu, dass es kein erstes Intervall mit lim #(n, n+1) / #(0, 1) = 0 geben kann, sondern dass das nur der Grenzwert für n --> oo ist. Das ist allerdings kein Beweis. Ich bin gespannt, ob jemand einen Beweis finden kann.

Gruß, WM
Ralf Bader
2020-06-13 19:16:19 UTC
Permalink
Post by Ganzhinterseher
Post by Juergen Ilse
Der Grenzwert existiert nicht (als "eigentlicher Grenzwert" bzw.
als "unei- gentlicher Grenzwert" waere er "unendlich". Beweis: Den
ersten Quotienten aus dem Intervall findet man erst bei q_5050, und
auch in den darauf folgenden 1000 Diagonalen findet man nur jeweils
ein Element aus diesem Intervall auf der jeweiligen Diagonale.
Damit steigt die Anzahl der Elemente aus dem Intervall ]0;1] auf
einer Diagonale linear mit der Nummer der jeweiligen Diagonale (bei
jeder zweiten Diagonale nimmt dieser Wert um 1 zu), bei den
Elementen aus dem Intervall ]1000;1001] nimmt die Anzahl der
Elemente aus diesem Intervall maximal alle 1000 Diagonalen um 1 zu
(und in der tausendsten Diagonale, die mit dem Element q_5050
beginnen wuerde waere erstmalig ein Element aus dem Intervall
]1000;1001] enthalten). Die Anzahl der Elemente aus dem Intervall
]0;1] steigen mit steigender Zahl der "durch- numerierten
Diagonalen" also sehr viel staerker an, als die Anzahl der Elemente
aus dem Intervall ]1000;1001].
Die Hälfte aller Brüche stammt aus dem ersten Intervall. Bezeichnen
wir sie mit #(0, 1). Also muss die andere Hälfte aus den anderen
Intervallen stammen. Also kann nicht jeder Grenzwert lim #(n, n+1) /
#(0, 1) = 0 sein. Stetigkeitsbetrachtungen führen mich dazu, dass es
kein erstes Intervall mit lim #(n, n+1) / #(0, 1) = 0 geben kann,
sondern dass das nur der Grenzwert für n --> oo ist. Das ist
allerdings kein Beweis. Ich bin gespannt, ob jemand einen Beweis
finden kann.
Gruß, WM
Da haben Sie ja direkt mal was hingekriegt, was nicht ganz falsch ist.
Dann will ich mal nicht so sein und verrate, daß der Anteil der Brüche
(nicht der rationalen Zahlen, also gekürzten Brüche) im Intervall
(i,i+1) (die beiden Endpunkte machen das Kraut weder mager noch fett)
gegen 1/(i^2+3i+2) strebt, wmls.
WM
2020-06-14 15:50:47 UTC
Permalink
Post by Ralf Bader
Post by Ganzhinterseher
Die Hälfte aller Brüche stammt aus dem ersten Intervall. Bezeichnen
wir sie mit #(0, 1). Also muss die andere Hälfte aus den anderen
Intervallen stammen. Also kann nicht jeder Grenzwert lim #(n, n+1) /
#(0, 1) = 0 sein. Stetigkeitsbetrachtungen führen mich dazu, dass es
kein erstes Intervall mit lim #(n, n+1) / #(0, 1) = 0 geben kann,
sondern dass das nur der Grenzwert für n --> oo ist. Das ist
allerdings kein Beweis. Ich bin gespannt, ob jemand einen Beweis
finden kann.
Dann will ich mal nicht so sein und verrate, daß der Anteil der Brüche
(nicht der rationalen Zahlen, also gekürzten Brüche) im Intervall
(i,i+1) (die beiden Endpunkte machen das Kraut weder mager noch fett)
gegen 1/(i^2+3i+2) strebt, wmls.
Die Endpunkte spielen kein Rolle, aber der Grenzwert sollte die Häufigkeit gekürzten Brüche angeben, denn der Bruch m/n käme sonst ja unendlich oft vor.

Gruß, WM
Juergen Ilse
2020-06-08 16:22:08 UTC
Permalink
Hallo,
Post by Juergen Ilse
Ach ja, und bevor ich es vergesse: selbst wenn dieser Grenzwert existiert,
besagt er (unabhaengig von seinem Wert) *nichts* darueber aus, ob die Can-
torsche Abbildung bisjektiv ist oder nicht. Auch wenn SIE jetzt wieder an-
fangen werden, irgendetwas mit angeblicher "Logik" daherzufasseln, um doch
einen Zusammenhang herzustellen: Nein, einen solchen Zusammenhang gibt es
*nicht*. Das die Abbildung bijektiv ist, ist aber leicht zu beweisen.
PS: Wenn der Grenzwert existieren sollte, liegt er mit Sicherheit >=1,
denn egal, wie weit man geht: Die Diagonale und alles oeberhalb der Diagonale
liegt mit Sicherheit *immer* im halboffenen Intervall ]0;1]. Egal, wie weit
man die Diagonalen durchzaehlt, ist also *immer* ungefaehr die Haelfte der
bis dahin abgezaehlten Elemente im halboffenen Intervall ]0;1] und damit mit
Sicherheit nicht im halboffenen Intervall ]1000;1001]. Darueber, ob die
Abbildung bijektiv ist oder nicht, laesst diese Betrachtung allerdings kei-
nerlei Aussage zu.
Der Grenzwert existiert nicht (als "eigentlicher Grenzwert" bzw. als "unei-
gentlicher Grenzwert" waere er "unendlich".
Beweis:
Den ersten Quotienten aus dem Intervall findet man erst bei q_5050, und auch
in den darauf folgenden 1000 Diagonalen findet man nur jeweils ein Element
aus diesem Intervall auf der jeweiligen Diagonale. Damit steigt die Anzahl
der Elemente aus dem Intervall ]0;1] auf einer Diagonale linear mit der Nummer
der jeweiligen Diagonale (bei jeder zweiten Diagonale nimmt dieser Wert um 1
zu), bei den Elementen aus dem Intervall ]1000;1001] nimmt die Anzahl der
Elemente aus diesem Intervall maximal alle 1000 Diagonalen um 1 zu (und in
der tausendsten Diagonale, die mit dem Element q_5050 beginnen wuerde waere
erstmalig ein Element aus dem Intervall ]1000;1001] enthalten). Die Anzahl
der Elemente aus dem Intervall ]0;1] steigen mit steigender Zahl der "durch-
numerierten Diagonalen" also sehr viel staerker an, als die Anzahl der
Elemente aus dem Intervall ]1000;1001].

Nur hat das nicht den geringsten Einfluss auf die Bijektivitaet der Abbildung
von der Menge der natuerlichen Zahlen auf die Menge der rationalen Zahlen,
warum sollte da auch ein Zusammenhang existieren?

Numeriert man im Gegensatz zu Cantor alle Diagonalen von links unten nach
rechts oben durch, kann man den Index ausrechnen, den der Bruch p/q in der
Abzaehlung hat:

Er stuende in der "p-ten Zeile" und "q-ten Spalte", damit also in der
"(p+q-1)-ten" Diagonale. Auf dieser Diagonale wuede er dann den "q-ten"
Platz belegen. Da die Summe aller Brueche in den ersten (p+q-2) Diagonalen
gleich (p+q-2)*(p+q-3)/2 ist (Summenformel fuer arichtmetische Reihen),
ergibt sich der Index des Bruches p/q zu q+(p+q-2)*(p+q-3)/2. Das ist mit
absoluter Sicherheit eine natuerliche Zahl. Ich kann also zu jedem positivn
Bruch bei *dieser* Abzaehlung genau den Index des Bruches bai der Abzaehlung
berechnen. Damit haben wir jetzt also sogar eine geschlossene Formel fuer
die bisjektive Abbildung der positiven Brueche auf die natuerlichen Zahlen:

p/q -> q+(p+q-2)*(p+q-3)/2

Dass diese Abbildung bijektiv ist, ergibt aich aus der Konstruktion der
Abbildung (es wird ja stets von links unten nach rechts oben an den Dia-
gonalen der Cantorschen Anordnung der Brueche "entlang gezaehlt").

Tschuess,
Juergen Ilse (***@usenet-verwaltung.de)
PS: Das ist noch keine bijektive Abbildung von den positiven rationalen
Zahlen auf die natuerlichen Zahlen, da wir ja rationale Zahlen mehrfach
zaehlen. Eine bijektive Abbildung wuerde sich nur dann ergeben, wenn man
die "nicht gekuerzten Brueche" auslassen wuerde, aber dann wuerde unsere
schoene oben stehende geschlossene Formel fuer die Abbildung nicht mehr
zutreffen ...
WM
2020-06-09 15:50:08 UTC
Permalink
Post by Juergen Ilse
Ach ja, und bevor ich es vergesse: selbst wenn dieser Grenzwert existiert,
besagt er (unabhaengig von seinem Wert) *nichts* darueber aus, ob die Can-
torsche Abbildung bisjektiv ist oder nicht.
Er sagt jedenfalls etwas über des Verhältnis der von Cantor nummerierten Brüche in den entsprechenden Intervallen aus, denn er betrifft ja alle verfügbaren Indizes.
Post by Juergen Ilse
Das die Abbildung bijektiv ist, ist aber leicht zu beweisen.
Es ist leicht zu beweisen, dass jeder definierbare Bruch indiziert wird.

Es ist aber ebenso leicht zu beweisen, dass jeder definierbare Index nichts über eine vollständig unendliche Menge beweist, denn

∀n ∈ ℕ_def: |ℕ \ {1, 2, 3, ..., n}| = ℵo.

Aber da machst Du ja einen Unterschied.
Post by Juergen Ilse
Egal, wie weit
man die Diagonalen durchzaehlt, ist also *immer* ungefaehr die Haelfte der
bis dahin abgezaehlten Elemente im halboffenen Intervall ]0;1] und damit mit
Sicherheit nicht im halboffenen Intervall ]1000;1001]. Darueber, ob die
Abbildung bijektiv ist oder nicht, laesst diese Betrachtung allerdings kei-
nerlei Aussage zu.
Es geht ja nur um die Frage, ob Cantor in (1000, 1001) genau so viele Brüche erwischt wie in (0, 1), wobei nicht die Bijektion betrachtet wird, denn die kann immer als gültig behauptet werden, solange man die Bedeutung der Gleichung

∀n ∈ ℕ_def: |ℕ \ {1, 2, 3, ..., n}| = ℵo

leugnet oder einfach nicht versteht, sondern das einfache Verhältnis.

Gruß, WM
WM
2020-06-09 15:41:33 UTC
Permalink
Post by Juergen Ilse
Post by Ganzhinterseher
Der Grenzwert oder Limes einer Folge von Zahlen ist eine Zahl, der die Folge beliebig nah kommt. (Wikipedia)
Das ist eine sehr "umgangssprachliche" und keine mathematisch exakte
Definition.
Die Darstellung mathematischer Symbole ist hier zu mühsam. Im Falle von echtem Interesse siehe: W. Mückenheim: "Mathematik für die ersten Semester", 4th ed., De Gruyter, Berlin (2015), S. 184.
Post by Juergen Ilse
Post by Ganzhinterseher
lim k --> oo [
|{x ∈ ℝ | 0 < x =< 1} ∩ {q_1, q_2, ..., q_k}|
/
|{x ∈ ℝ | 1000 < x =< 1001} ∩ {q_1, q_2, ..., q_k}|]
wobei die q_i der Cantorschen Folge
1/1, 1/2, 2/1, 1/3, 3/1, 1/4, 2/3, 3/2, 4/1, 1/5, 5/1, 1/6, ...
entnommen sind.
Wenn man einen Grenzwert bestimmen will, muss man zuerst einmal nachweisen,
dass dieser Grenzwert ueberhaupt existiert oder es sich moeglicherweise um
eine divergente Folge handelt.
Wenn man ihn schon bestimmt hat, so braucht man diesen Beweis nicht mehr.
Post by Juergen Ilse
Wo ist IHR Beweis der Existenz eines Grenz-
wertes )weil ohne einen solchen Nachweis die Frage nac hdem Grenzwert selbst
voellig unsinnig waere).
Falls kein Grenzwert existiert, sondern zum Beispiel mehrere Häufungspunkte, so wäre ich mit der Bestimmung eines einzigen schon zufrieden.

Gruß, WM
Juergen Ilse
2020-06-10 22:40:23 UTC
Permalink
Hallo,
Post by WM
Post by Juergen Ilse
Wenn man einen Grenzwert bestimmen will, muss man zuerst einmal nachweisen,
dass dieser Grenzwert ueberhaupt existiert oder es sich moeglicherweise um
eine divergente Folge handelt.
Wenn man ihn schon bestimmt hat, so braucht man diesen Beweis nicht mehr.
SIE haben aber weder die Existenz des Grenzwertes bewiesen noch einen moeg-
lichen Wert fuer den Grenzwert bestimmt. Ausser unbewiesenen Behauptungen
ueber einen niicht existierenden zusammenhang zwischen einem solchen Grenzwert
und der Bijektion der cantorschen Abbildung kam von iHNEN gar nichts.
Post by WM
Post by Juergen Ilse
Wo ist IHR Beweis der Existenz eines Grenz-
wertes )weil ohne einen solchen Nachweis die Frage nac hdem Grenzwert selbst
voellig unsinnig waere).
Falls kein Grenzwert existiert, sondern zum Beispiel mehrere Häufungspunkte, so wäre ich mit der Bestimmung eines einzigen schon zufrieden.
Dann beweisen SIE die Existenz eines solchen Haeufungspunkts und geben SIE
den Wert eines solchen an. Aber das koennen SIE natuerlich nicht, denn dazu
reichen IHRE mathematischen Kenntnisse noch nicht einmal im Ansatz aus ...

Vielleicht versuchen SiE es dann mal mit einem Gegenbeweis zur von mir
bewiesenen Bijektivitet der von mir genannten (gegenueber cantors Vorschlag
modifizierten) Abbildung der positiven Brueche auf die natuerlichen Zahlen.
Ich hatte sogar eine geschlossene Formel fuer das Bild jeden Bruchs angege-
ben. Also versuchen SIE es doch mit einem *BEWEIS*, dass diese Abbildung
nicht bijektiv ist ... Aber auch das wird IHNEN nicht gelingen.

Tschuess,
Juergegn Ilse (***@usenet-verwaltung.de)
Ganzhinterseher
2020-06-11 14:21:46 UTC
Permalink
Post by Juergen Ilse
Hallo,
Post by WM
Post by Juergen Ilse
Wenn man einen Grenzwert bestimmen will, muss man zuerst einmal nachweisen,
dass dieser Grenzwert ueberhaupt existiert oder es sich moeglicherweise um
eine divergente Folge handelt.
Wenn man ihn schon bestimmt hat, so braucht man diesen Beweis nicht mehr.
SIE haben aber weder die Existenz des Grenzwertes bewiesen noch einen moeg-
lichen Wert fuer den Grenzwert bestimmt.
Deswegen frage ich ja. Wenn jemand einen Grenzwert angeben kann, dann brauche ich nicht mehr zu beweisen, dass es ihn gibt.
Post by Juergen Ilse
Post by WM
Falls kein Grenzwert existiert, sondern zum Beispiel mehrere Häufungspunkte, so wäre ich mit der Bestimmung eines einzigen schon zufrieden.
Dann beweisen SIE die Existenz eines solchen Haeufungspunkts und geben SIE
den Wert eines solchen an.
Wenn einen unendlichen Folge beschränkt ist, so folgt die Existenz von Häufungspunkten.

Die Folge
[|{x ∈ ℝ | 1000 < x =< 1001} ∩ {q_1, q_2, ..., q_k}|/
|{x ∈ ℝ | 0 < x =< 1} ∩ {q_1, q_2, ..., q_k}|]
ist beschränkt durch 0 und 1.

Also besitzt sie mindestens einen Häufungspunkt.
Post by Juergen Ilse
Vielleicht versuchen SiE es dann mal mit einem Gegenbeweis zur von mir
bewiesenen Bijektivitet der von mir genannten (gegenueber cantors Vorschlag
modifizierten) Abbildung der positiven Brueche auf die natuerlichen Zahlen.
Ich hatte sogar eine geschlossene Formel fuer das Bild jeden Bruchs angege-
ben. Also versuchen SIE es doch mit einem *BEWEIS*, dass diese Abbildung
nicht bijektiv ist ... Aber auch das wird IHNEN nicht gelingen.
Das ist sogar ganz einfach. So einfach, dass jeder nicht durch die Mengenlehre geschädigte Mensch das erkennt.

Nach Cantor besitzt der Radius eines Quarks genau so viele rationale Punkte wie das gesamte unendliche Universum. Das ist so hirnrissig, dass nur ein orthodoxer Matheologe es glauben kann.

Selbstverständlich beruht Cantors Messung der abzählbaren Menge darauf, dass er jeweils nur einen winzigen, verschwindend kleinen Teil der Mengen erfasst. Das hat er ja sogar selbst erkannt:

∀n ∈ ℕ_erfasst: |ω| - n = |ω|

Also wird unendlich vieles nicht erfasst. Und das erklärt die Gleichmächtigkeit sofort. Wenn man aus riesigen Bergen und kleinen Resten Kaffeesatz immer nur ein winziges Körnchen entnimmt, dann ist es kein Wunder, dass alle "gleichmächtig" erscheinen.

Gruß, WM
Mostowski Collapse
2020-06-11 15:52:43 UTC
Permalink
WM haluziniert: "alle "gleichmächtig" erscheinen."

Es ist nicht alles gleichmächtig z.B. |Q| < |R|.
Q is abzählbar, und R ist überabzählbar.

Sind Sie jetzt zufrieden? Oder passt Ihnen das
auch nicht. Entscheiden Sie sich bitte.
Post by Ganzhinterseher
Post by Juergen Ilse
Hallo,
Post by WM
Post by Juergen Ilse
Wenn man einen Grenzwert bestimmen will, muss man zuerst einmal nachweisen,
dass dieser Grenzwert ueberhaupt existiert oder es sich moeglicherweise um
eine divergente Folge handelt.
Wenn man ihn schon bestimmt hat, so braucht man diesen Beweis nicht mehr.
SIE haben aber weder die Existenz des Grenzwertes bewiesen noch einen moeg-
lichen Wert fuer den Grenzwert bestimmt.
Deswegen frage ich ja. Wenn jemand einen Grenzwert angeben kann, dann brauche ich nicht mehr zu beweisen, dass es ihn gibt.
Post by Juergen Ilse
Post by WM
Falls kein Grenzwert existiert, sondern zum Beispiel mehrere Häufungspunkte, so wäre ich mit der Bestimmung eines einzigen schon zufrieden.
Dann beweisen SIE die Existenz eines solchen Haeufungspunkts und geben SIE
den Wert eines solchen an.
Wenn einen unendlichen Folge beschränkt ist, so folgt die Existenz von Häufungspunkten.
Die Folge
[|{x ∈ ℝ | 1000 < x =< 1001} ∩ {q_1, q_2, ..., q_k}|/
|{x ∈ ℝ | 0 < x =< 1} ∩ {q_1, q_2, ..., q_k}|]
ist beschränkt durch 0 und 1.
Also besitzt sie mindestens einen Häufungspunkt.
Post by Juergen Ilse
Vielleicht versuchen SiE es dann mal mit einem Gegenbeweis zur von mir
bewiesenen Bijektivitet der von mir genannten (gegenueber cantors Vorschlag
modifizierten) Abbildung der positiven Brueche auf die natuerlichen Zahlen.
Ich hatte sogar eine geschlossene Formel fuer das Bild jeden Bruchs angege-
ben. Also versuchen SIE es doch mit einem *BEWEIS*, dass diese Abbildung
nicht bijektiv ist ... Aber auch das wird IHNEN nicht gelingen.
Das ist sogar ganz einfach. So einfach, dass jeder nicht durch die Mengenlehre geschädigte Mensch das erkennt.
Nach Cantor besitzt der Radius eines Quarks genau so viele rationale Punkte wie das gesamte unendliche Universum. Das ist so hirnrissig, dass nur ein orthodoxer Matheologe es glauben kann.
∀n ∈ ℕ_erfasst: |ω| - n = |ω|
Also wird unendlich vieles nicht erfasst. Und das erklärt die Gleichmächtigkeit sofort. Wenn man aus riesigen Bergen und kleinen Resten Kaffeesatz immer nur ein winziges Körnchen entnimmt, dann ist es kein Wunder, dass alle "gleichmächtig" erscheinen.
Gruß, WM
Juergen Ilse
2020-06-11 17:16:02 UTC
Permalink
Hallo,
Post by Ganzhinterseher
Nach Cantor besitzt der Radius eines Quarks genau so viele rationale Punkte wie das gesamte unendliche Universum. Das ist so hirnrissig, dass nur ein orthodoxer Matheologe es glauben kann.
Falsch, denn Cantor rechnet eben bei unendlichen Mengen nicht mit "Anzahlen
#von Elementen" sondern mit "Maechtigkeiten von Mengen". Letztere hat er
genau *deswegen* eingefuehrt, weilman bei unendlichen Mengen nicht sinnvoll
mit "Anzahlen von Elementen" arbeiten kann. Maechtigkeiten sind keine
"Anzahlen", deswegen ist ihre bloedsinnige Argumentation auch nicht schluessig.

Tschuess,
Juergen Ilse (***@usenet-verwaltung.de)
Ganzhinterseher
2020-06-11 18:51:36 UTC
Permalink
Post by Juergen Ilse
Hallo,
Post by Ganzhinterseher
Nach Cantor besitzt der Radius eines Quarks genau so viele rationale Punkte wie das gesamte unendliche Universum. Das ist so hirnrissig, dass nur ein orthodoxer Matheologe es glauben kann.
Falsch, denn Cantor rechnet eben bei unendlichen Mengen nicht mit "Anzahlen
#von Elementen"
Doch, genau das tut er. Sonst würde auch das Abzählen ein verfehltes Wort sein.


Fasse ich das Unendliche so auf, wie dies von mir hier und bei meinen früheren Versuchen geschehen ist, so folgt daraus für mich ein wahrer Genuß, dem ich mich dankerfüllt hingebe, zu sehen, wie der ganze Zahlbegriff, der im Endlichen nur den Hintergrund der Anzahl hat, wenn wir aufsteigen zum Unendlichen, sich gewissermaßen spaltet in zwei Begriffe, in denjenigen der Mächtigkeit, welche unabhängig ist von der Ordnung, die einer Menge gegeben wird, und in den der Anzahl, welche notwendig an eine gesetzmäßige Ordnung der Menge gebunden ist, vermöge welcher letztere zu einer wohlgeordneten Menge wird.

Von Bedeutung ist es, daß die Ordnungszahlen stets auch in unendlicher Anzahl sich summieren lassen, so daß ihre Summe eine bestimmte, von der Reihenfolge ihrer Summanden abhängige Ordnungszahl ist.

letzteres stimmt überein mit dem, was ich früher (Grundl. e. allg. Mannigfaltigkeitslehre) [III 4, S. 168] "Anzahl einer wohlgeordneten Menge" genannt habe.

Ein anderes ist es, was ich "Anzahl" oder "Ordnungszahl" nenne; ich schreibe sie nur "wohlgeordneten Mengen" zu, und zwar verstehe ich unter der "Anzahl oder Ordnungszahl einer wohlgeordneten Menge" denjenigen Allgemeinbegriff, unter welchen alle wohlgeordneten Mengen, welche der gegebenen ähnlich sind, und nur diese fallen.

sondern mit "Maechtigkeiten von Mengen". Letztere hat er
Post by Juergen Ilse
genau *deswegen* eingefuehrt, weilman bei unendlichen Mengen nicht sinnvoll
mit "Anzahlen von Elementen" arbeiten kann.
Nein, er hat sie deswegen eingeführt, weil er bemerkt hat, dass sich Widersprüche ergeben, die man mit Mächtigkeiten notdürftig kaschieren kann.
Post by Juergen Ilse
Maechtigkeiten sind keine
"Anzahlen", deswegen
ist der Begriff sinnlos.

Gruß, WM
Mostowski Collapse
2020-06-11 19:41:42 UTC
Permalink
Das Abzählen überlebt im Wort "abzählbar".
Bedeutet aber nur endlich oder unendlich und
gleichmächtig wie ω.

Aber wie man je weiss gibt es auch "überabzählbare"
Mengen, das bedeutet dann unendlich und
mächtiger als ω.

Beides hat Cantor untersucht. Man kann ihm also nicht
vorwerfen, dass er auf abzählbar eingeschossen ist.
Post by Ganzhinterseher
Post by Juergen Ilse
Hallo,
Post by Ganzhinterseher
Nach Cantor besitzt der Radius eines Quarks genau so viele rationale Punkte wie das gesamte unendliche Universum. Das ist so hirnrissig, dass nur ein orthodoxer Matheologe es glauben kann.
Falsch, denn Cantor rechnet eben bei unendlichen Mengen nicht mit "Anzahlen
#von Elementen"
Doch, genau das tut er. Sonst würde auch das Abzählen ein verfehltes Wort sein.
Fasse ich das Unendliche so auf, wie dies von mir hier und bei meinen früheren Versuchen geschehen ist, so folgt daraus für mich ein wahrer Genuß, dem ich mich dankerfüllt hingebe, zu sehen, wie der ganze Zahlbegriff, der im Endlichen nur den Hintergrund der Anzahl hat, wenn wir aufsteigen zum Unendlichen, sich gewissermaßen spaltet in zwei Begriffe, in denjenigen der Mächtigkeit, welche unabhängig ist von der Ordnung, die einer Menge gegeben wird, und in den der Anzahl, welche notwendig an eine gesetzmäßige Ordnung der Menge gebunden ist, vermöge welcher letztere zu einer wohlgeordneten Menge wird.
Von Bedeutung ist es, daß die Ordnungszahlen stets auch in unendlicher Anzahl sich summieren lassen, so daß ihre Summe eine bestimmte, von der Reihenfolge ihrer Summanden abhängige Ordnungszahl ist.
letzteres stimmt überein mit dem, was ich früher (Grundl. e. allg. Mannigfaltigkeitslehre) [III 4, S. 168] "Anzahl einer wohlgeordneten Menge" genannt habe.
Ein anderes ist es, was ich "Anzahl" oder "Ordnungszahl" nenne; ich schreibe sie nur "wohlgeordneten Mengen" zu, und zwar verstehe ich unter der "Anzahl oder Ordnungszahl einer wohlgeordneten Menge" denjenigen Allgemeinbegriff, unter welchen alle wohlgeordneten Mengen, welche der gegebenen ähnlich sind, und nur diese fallen.
sondern mit "Maechtigkeiten von Mengen". Letztere hat er
Post by Juergen Ilse
genau *deswegen* eingefuehrt, weilman bei unendlichen Mengen nicht sinnvoll
mit "Anzahlen von Elementen" arbeiten kann.
Nein, er hat sie deswegen eingeführt, weil er bemerkt hat, dass sich Widersprüche ergeben, die man mit Mächtigkeiten notdürftig kaschieren kann.
Post by Juergen Ilse
Maechtigkeiten sind keine
"Anzahlen", deswegen
ist der Begriff sinnlos.
Gruß, WM
Ganzhinterseher
2020-06-12 09:41:05 UTC
Permalink
Post by Mostowski Collapse
Das Abzählen überlebt im Wort "abzählbar".
Bedeutet aber nur endlich oder unendlich und
gleichmächtig wie ω.
Es bedeutet, dass eine Bijektion existiert, also jeder Punkt aus Menge A einen und nur einen Partner aus Menge B besitzt. Das kann man zum Beispiel auf die (rationalen oder reellen) Punkte im Durchmesser (ode sogar Radius) eines Quarks und im unendlichen 4-dimensionalen Universum anwenden und "beweisen", dass in beiden Mengen gleichviele Punkte existieren - nicht ein einziger mehr oder weniger. Diese Methode liefert also ein Ergebnis, das so falsch ist wie nur irgendmöglich. Folglich kann man schließen, dass nicht alle Punkte erfasst werden, sondern nur ein kleiner Bruchteil - und zwar jeweils von beiden Mengen.
Post by Mostowski Collapse
Beides hat Cantor untersucht. Man kann ihm also nicht
vorwerfen, dass er auf abzählbar eingeschossen ist.
Man kann ihm vorwerfen, dass er die Mathematiker bestimmt hat, einen sinnlosen Begriff anzuwenden.

Im Übrigen sind Cantors Ordinal-"Zahlen" auch abgesehen davon ohnehin verfehlt, denn sie hängen von der Anordnung ab, was schon Frege (leider erfolglos) kritisiert hat.

Gruß, WM
Me
2020-06-12 11:37:56 UTC
Permalink
Im Übrigen sind Cantors Ordinal-"Zahlen" auch abgesehen davon ohnehin ver-
fehlt, denn sie hängen von der Anordnung ab, was schon Frege (leider erfolg-
los) kritisiert hat.
Ach, halt doch mal die Klappe, Mann. Frege hat nicht Cantors Ordinalzahl-Begriff kritisiert, sondern lediglich den Umstand, dass Cantor dafür das Wort "Anzahl" verwendet hat. (Für wie blöde halten Sie eigentlich die Leser Ihres Krampfgefasels, Mückenheim?)

"§ 85. Die cantorschen unendlichen Anzahlen; "Mächtigkeit". Abweichung in
der Benennung.

Vor Kurzem hat G. Cantor in einer bemerkenswerthen Schrift unendliche
Anzahlen eingeführt. Ich stimme ihm durchaus in der Würdigung der Ansicht
bei, welche überhaupt nur die endlichen Anzahlen als wirklich gelten lassen
will. Sinnlich wahrnehmbar und räumlich sind weder diese noch die Brüche,
noch die negativen, irrationalen und complexen Zahlen; und wenn man
wirklich nennt, was auf die Sinne wirkt, oder was wenigstens Wirkungen hat,
die Sinneswahrnehmungen zur nähern oder entferntern Folge haben können, so
ist freilich keine dieser Zahlen wirklich. Aber wir brauchen auch solche
Wahrnehmungen gar nicht als Beweisgründe für unsere Lehrsätze. Einen Namen
oder ein Zeichen, das logisch einwurfsfrei eingeführt ist, können wir in
unsern Untersuchungen ohne Scheu gebrauchen, und so ist unsere Anzahl oo_1
so gerechtfertigt wie die Zwei oder die Drei.
Indem ich hierin, wie ich glaube, mit Cantor übereinstimme, weiche ich doch
in der Benennung etwas von ihm ab. Meine Anzahl nennt er "Mächtigkeit,"
während sein Begriff der Anzahl auf die Anordnung Bezug nimmt. Für endliche
Anzahlen ergiebt sich freilich doch eine Unabhängigkeit von der
Reihenfolge, dagegen nicht für unendlichgrosse. Nun enthält der
Sprachgebrauch des Wortes "Anzahl" und der Frage "wieviele?" keine
Hinweisung auf eine bestimmte Anordnung. Cantors Anzahl antwortet vielmehr
auf die Frage: "das wievielste Glied in der Succession ist das Endglied?"
Darum scheint mir meine Benennung besser mit dem Sprachgebrauche
übereinzustimmen. Wenn man die Bedeutung eines Wortes erweitert, so wird
man darauf zu achten haben, dass möglichst viele allgemeine Sätze ihre
Geltung behalten und zumal so grundlegende, wie für die Anzahl die
Unabhängigkeit von der Reihenfolge ist. Wir haben gar keine Erweiterung
nöthig gehabt, weil unser Begriff der Anzahl sofort auch unendliche Zahlen
umfasst."

(Gottlob Frege, Grundlagen der Arithmetik, 1884)
Mostowski Collapse
2020-06-12 18:15:19 UTC
Permalink
Bei unendlichen Mengen kann es ein einzelner mehr
oder wenige schon sein. Das wusste schon Dedekind,
hat er unendliche Menge doch definiert, als

eine Menge die sich auf einen echten Teil sich selbers
abbilden lässt. Also es ist nie die Rede "nicht ein
einziger mehr oder weniger".

Das ist Ihre falsche Interpretation.

https://de.wikipedia.org/wiki/Unendliche_Menge#Dedekind-Unendlichkeit
Post by Ganzhinterseher
Post by Mostowski Collapse
Das Abzählen überlebt im Wort "abzählbar".
Bedeutet aber nur endlich oder unendlich und
gleichmächtig wie ω.
Es bedeutet, dass eine Bijektion existiert, also jeder Punkt aus Menge A einen und nur einen Partner aus Menge B besitzt. Das kann man zum Beispiel auf die (rationalen oder reellen) Punkte im Durchmesser (ode sogar Radius) eines Quarks und im unendlichen 4-dimensionalen Universum anwenden und "beweisen", dass in beiden Mengen gleichviele Punkte existieren - nicht ein einziger mehr oder weniger. Diese Methode liefert also ein Ergebnis, das so falsch ist wie nur irgendmöglich. Folglich kann man schließen, dass nicht alle Punkte erfasst werden, sondern nur ein kleiner Bruchteil - und zwar jeweils von beiden Mengen.
Post by Mostowski Collapse
Beides hat Cantor untersucht. Man kann ihm also nicht
vorwerfen, dass er auf abzählbar eingeschossen ist.
Man kann ihm vorwerfen, dass er die Mathematiker bestimmt hat, einen sinnlosen Begriff anzuwenden.
Im Übrigen sind Cantors Ordinal-"Zahlen" auch abgesehen davon ohnehin verfehlt, denn sie hängen von der Anordnung ab, was schon Frege (leider erfolglos) kritisiert hat.
Gruß, WM
Juergen Ilse
2020-06-12 12:23:43 UTC
Permalink
Hallo,
Post by Ganzhinterseher
Post by Juergen Ilse
Post by Ganzhinterseher
Nach Cantor besitzt der Radius eines Quarks genau so viele rationale Punkte wie das gesamte unendliche Universum. Das ist so hirnrissig, dass nur ein orthodoxer Matheologe es glauben kann.
Falsch, denn Cantor rechnet eben bei unendlichen Mengen nicht mit "Anzahlen
von Elementen"
Doch, genau das tut er.
Nein. Da er erkannt hat, dass der Begriff "Anzahl" bei unendlichen Mengen
*sinnlos* ist, hat er den Begriff der "Maechtigkeit" etabliert, der fuer
endliche Mengen mit "AAnzahl der Elemente" uebereinstimmt, aber auch bei
unendlichen Mengen Sinn ergibt (ganz imGegensatz zum Begriss "Anzahl der
Elemente").
Post by Ganzhinterseher
Fasse ich das Unendliche so auf, wie dies von mir hier und bei meinen früheren Versuchen geschehen ist, so folgt daraus für mich ein wahrer Genuß, dem ich mich dankerfüllt hingebe, zu sehen, wie der ganze Zahlbegriff, der im Endlichen nur den Hintergrund der Anzahl hat, wenn wir aufsteigen zum Unendlichen, sich gewissermaßen spaltet in zwei Begriffe, in denjenigen der Mächtigkeit, welche unabhängig ist von der Ordnung, die einer Menge gegeben wird, und in den der Anzahl, welche notwendig an eine gesetzmäßige Ordnung der Menge gebunden ist, vermöge welcher letztere zu einer wohlgeordneten Menge wird.
Ersteres ist (wie er schrieb) die Maechtigkeit, letzterer der Begriff der
"Ordinalzahl", der aber genau genommen bei (ungeordneten) Mengen Bedeutungs-
los ist.

Nur weil SIE unfaehig sind, den Begriff der Maechtigkeit und seine Bedeutung
zu begreifen, ist der Begriff der Maechtigkeit noch lange nicht sinnlos.

Tschuess,
Juergen Ilse (***@usenet-verwaltung.de)
Me
2020-06-12 12:48:29 UTC
Permalink
Post by Juergen Ilse
Post by Juergen Ilse
Cantor rechnet eben bei unendlichen Mengen nicht mit "Anzahlen
von Elementen"
Doch. Nur muss man wissen, dass Cantor die Ordinalzahlen als "Anzahlen" bezeichnet hat.
Post by Juergen Ilse
Nein. Da er erkannt hat, dass der Begriff "Anzahl" bei unendlichen Mengen
*sinnlos* ist,
Nein, so etwas hat er nicht "erkannt" (jedenfalls -meines Wissens nach- nie zum Ausdruck gebracht).
Post by Juergen Ilse
hat er den Begriff der "Maechtigkeit" etabliert, der fuer
endliche Mengen mit "Anzahl der Elemente" uebereinstimmt,
In der Tat.
Post by Juergen Ilse
aber auch bei unendlichen Mengen Sinn ergibt
Ebenso wie der Begriff "Anzahl", der bei Cantor allerdings (anders als bei Frege) "auf die Anordnung" Bezug nimmt.
Post by Juergen Ilse
[...] zu sehen, wie der ganze Zahlbegriff, der im Endlichen nur den
Hintergrund der Anzahl hat, wenn wir aufsteigen zum Unendlichen, sich
gewissermaßen spaltet in zwei Begriffe, in denjenigen der Mächtigkeit,
welche unabhängig ist von der Ordnung, die einer Menge gegeben wird,
und in den der Anzahl, welche notwendig an eine gesetzmäßige Ordnung
der Menge gebunden ist, vermöge welcher letztere zu einer wohlgeordneten
Menge wird.
Ersteres ist (wie er schrieb) die Maechtigkeit, letzterer der Begriff der
"Ordinalzahl"
So ist es. Cantor hat aber in diesem Zusammenhang (wie WM richtig bemerkt) tatsächlich von "Anzahl" gesprochen.

"§ 85. Die cantorschen unendlichen Anzahlen; "Mächtigkeit". Abweichung in
der Benennung.

Vor Kurzem hat G. Cantor in einer bemerkenswerthen Schrift unendliche
Anzahlen eingeführt. Ich stimme ihm durchaus in der Würdigung der Ansicht
bei, welche überhaupt nur die endlichen Anzahlen als wirklich gelten lassen
will. Sinnlich wahrnehmbar und räumlich sind weder diese noch die Brüche,
noch die negativen, irrationalen und complexen Zahlen; und wenn man
wirklich nennt, was auf die Sinne wirkt, oder was wenigstens Wirkungen hat,
die Sinneswahrnehmungen zur nähern oder entferntern Folge haben können, so
ist freilich keine dieser Zahlen wirklich. Aber wir brauchen auch solche
Wahrnehmungen gar nicht als Beweisgründe für unsere Lehrsätze. Einen Namen
oder ein Zeichen, das logisch einwurfsfrei eingeführt ist, können wir in
unsern Untersuchungen ohne Scheu gebrauchen, und so ist unsere Anzahl oo_1
so gerechtfertigt wie die Zwei oder die Drei.
Indem ich hierin, wie ich glaube, mit Cantor übereinstimme, weiche ich doch
in der Benennung etwas von ihm ab. Meine Anzahl nennt er "Mächtigkeit,"
während sein Begriff der Anzahl auf die Anordnung Bezug nimmt. Für endliche
Anzahlen ergiebt sich freilich doch eine Unabhängigkeit von der
Reihenfolge, dagegen nicht für unendlichgrosse. Nun enthält der
Sprachgebrauch des Wortes "Anzahl" und der Frage "wieviele?" keine
Hinweisung auf eine bestimmte Anordnung. Cantors Anzahl antwortet vielmehr
auf die Frage: "das wievielste Glied in der Succession ist das Endglied?"
Darum scheint mir meine Benennung besser mit dem Sprachgebrauche
übereinzustimmen. Wenn man die Bedeutung eines Wortes erweitert, so wird
man darauf zu achten haben, dass möglichst viele allgemeine Sätze ihre
Geltung behalten und zumal so grundlegende, wie für die Anzahl die
Unabhängigkeit von der Reihenfolge ist. Wir haben gar keine Erweiterung
nöthig gehabt, weil unser Begriff der Anzahl sofort auch unendliche Zahlen
umfasst."

(Gottlob Frege, Grundlagen der Arithmetik, 1884)
Juergen Ilse
2020-06-12 13:14:24 UTC
Permalink
Hallo,
Post by Me
Post by Juergen Ilse
Nein. Da er erkannt hat, dass der Begriff "Anzahl" bei unendlichen Mengen
*sinnlos* ist,
Nein, so etwas hat er nicht "erkannt" (jedenfalls -meines Wissens nach- nie zum Ausdruck gebracht).
Doch.
Post by Me
Post by Juergen Ilse
hat er den Begriff der "Maechtigkeit" etabliert, der fuer
endliche Mengen mit "Anzahl der Elemente" uebereinstimmt,
In der Tat.
Post by Juergen Ilse
aber auch bei unendlichen Mengen Sinn ergibt
Ebenso wie der Begriff "Anzahl", der bei Cantor allerdings (anders als bei
Frege) "auf die Anordnung" Bezug nimmt.
Eben. Bei Mengen ohne weitere Struktur (also insbesondere ohne Ordnung
bzw. Reihenfolge) ist der Begriff der Ordinalzahl sinnlos, dort ist allein
die Maechtigkeit ein sinnvolles Mass fuer einen "Groessenvergleich von
Mengen".
Post by Me
Post by Juergen Ilse
Ersteres ist (wie er schrieb) die Maechtigkeit, letzterer der Begriff der
"Ordinalzahl"
So ist es. Cantor hat aber in diesem Zusammenhang (wie WM richtig bemerkt) tatsächlich von "Anzahl" gesprochen.
Aber nicht fuer (ungeordnete) Mengen, sondern nur fuer Mengen, die
*zusaetzlich* mit irgend einerArt von Ordnung versehen wurden.

Tschuess,
Juergen Ilse (***@usenet-verwaltung.de)
Me
2020-06-12 14:17:01 UTC
Permalink
Post by Juergen Ilse
Post by Me
Post by Juergen Ilse
Nein. Da er erkannt hat, dass der Begriff "Anzahl" bei unendlichen Mengen
*sinnlos* ist
Nein, so etwas hat er nicht "erkannt" (jedenfalls -meines Wissens nach- nie
zum Ausdruck gebracht).
Doch.
Mach hier doch nicht den Mückenheim. :-P

Kannst Du Deine Behauptung durch ein entsprechendes Zitat Cantors belegen? Nein? Dann halt Dich mit diesen "Mutmaßungen" (man kann auch von Hirngespinsten sprechen) bitte etwas zurück.

Tatsächlich wird trifft es das folgende wohl besser:

"Ordinalzahlen erlauben die Verallgemeinerung der auf Folgen beschränkten Beweisverfahren der vollständigen Induktion auf beliebig große Mengen oder auch echte Klassen, sofern sie sich wohlordnen lassen, mittels des Verfahrens der transfiniten Induktion.

Die Beschreibung der Größe einer Menge, naiv gesprochen der Anzahl ihrer Elemente, führt im Gegensatz dazu zu dem Begriff Kardinalzahl (eins, zwei, drei, ...).

Georg Cantor hatte die Idee, wie man die beiden Konzepte – Zahl als Größe und Zahl als Index – innerhalb der Mengenlehre auf unendliche Mengen verallgemeinern kann; denn während sie für endliche Mengen übereinstimmen, muss man sie für unendliche Mengen unterscheiden. Kardinalzahlen werden dabei als spezielle Ordinalzahlen definiert."

Es kann also KEINE REDE DAVON SEIN, dass Cantor "erkannt hat, dass der Begriff "Anzahl" bei unendlichen Mengen *sinnlos* ist".
Post by Juergen Ilse
Post by Me
Post by Juergen Ilse
er hat den Begriff der "Maechtigkeit" etabliert, der fuer
endliche Mengen mit "Anzahl der Elemente" [so wie Cantor ihn
verstanden hat --Me] uebereinstimmt,
In der Tat.
Frege dazu:

"Indem ich hierin, wie ich glaube, mit Cantor übereinstimme, weiche ich doch
in der Benennung etwas von ihm ab. Meine Anzahl nennt er "Mächtigkeit,"
während sein Begriff der Anzahl auf die Anordnung Bezug nimmt.
Für endliche Anzahlen ergiebt sich freilich doch eine Unabhängigkeit von der
Reihenfolge, dagegen nicht für unendlichgrosse."
Post by Juergen Ilse
Bei Mengen ohne weitere Struktur (also insbesondere ohne Ordnung
bzw. Reihenfolge) ist der Begriff der Ordinalzahl sinnlos, dort
ist allein die Maechtigkeit ein sinnvolles Mass fuer einen "Groessen-
vergleich von Mengen".
Mag sein. ANDERERSEITS kann a) jede Menge wohlgeordnet werden (!) und b) kann man die (unendlichen) Kardinalzahlen auch mithilfe der Ordinalzahltheorie begründen (was auch vielfach so gemacht wird).
Post by Juergen Ilse
Post by Me
Cantor hat aber in diesem Zusammenhang (wie WM richtig bemerkt) tatsächlich
von "Anzahl" gesprochen.
Aber nicht fuer (ungeordnete) Mengen, sondern nur fuer Mengen, die
*zusaetzlich* mit irgend einer Art von Ordnung versehen wurden.
Natürlich, wie könnte es auch anders sein? Eben DARUM hat Frege ja auch Einspruch gegen die Verwendung des Wortes "Anzahl" (für Ordinalzahlen) erhoben:

"Indem ich [sachlich -me], wie ich glaube, mit Cantor übereinstimme, weiche ich doch
in der Benennung etwas von ihm ab. Meine Anzahl nennt er "Mächtigkeit,"
während sein Begriff der Anzahl auf die Anordnung Bezug nimmt. Für endliche
Anzahlen ergiebt sich freilich doch eine Unabhängigkeit von der
Reihenfolge, dagegen nicht für unendlichgrosse. Nun enthält der
Sprachgebrauch des Wortes "Anzahl" und der Frage "wieviele?" keine
Hinweisung auf eine bestimmte Anordnung. Cantors Anzahl antwortet vielmehr
auf die Frage: "das wievielste Glied in der Succession ist das Endglied?"
Darum scheint mir meine Benennung besser mit dem Sprachgebrauche
übereinzustimmen. Wenn man die Bedeutung eines Wortes erweitert, so wird
man darauf zu achten haben, dass möglichst viele allgemeine Sätze ihre
Geltung behalten und zumal so grundlegende, wie für die Anzahl die
Unabhängigkeit von der Reihenfolge ist. Wir haben gar keine Erweiterung
nöthig gehabt, weil unser Begriff der Anzahl sofort auch unendliche Zahlen
umfasst."

(Gottlob Frege, Grundlagen der Arithmetik, 1884)

Vielleicht ist es sinnvoll in diesem Zusammenhang zu bemerken, dass Cantor den "Wohlordnungssatz" (also dass jede Menge wohlgeordnet werden kann) für eine "Denknotwendigkeit" hielt.

Siehe dazu: https://books.google.de/books?id=YHyVDwAAQBAJ&pg=PA161&lpg=PA161&dq=Wohlordnungssatz+Cantor+denknotwendigkeit&source=bl&ots=xgsncUp_y4&sig=ACfU3U22ojpPDhcqR_3RnYfbpYj5cRFnNQ&hl=en&sa=X&ved=2ahUKEwjn26nFufzpAhWGzKQKHWgtArEQ6AEwAXoECAoQAQ#v=onepage&q=Wohlordnungssatz%20Cantor%20denknotwendigkeit&f=false
Juergen Ilse
2020-06-12 18:35:31 UTC
Permalink
Hallo,
Post by Me
Georg Cantor hatte die Idee, wie man die beiden Konzepte – Zahl als Größe und Zahl als Index – innerhalb der Mengenlehre auf unendliche Mengen verallgemeinern kann; denn während sie für endliche Mengen übereinstimmen, muss man sie für unendliche Mengen unterscheiden. Kardinalzahlen werden dabei als spezielle Ordinalzahlen definiert."
Eben.
Post by Me
Es kann also KEINE REDE DAVON SEIN, dass Cantor "erkannt hat, dass der Begriff "Anzahl" bei unendlichen Mengen *sinnlos* ist".
Doch, eben *weil* er die Unterscheidung getroffen hat. Die Ordinalzahlen
sind fuer "Mengen im allgemeinen" unbrauchbar, denn sie bedingen eine
Ordnung auf der Menge, die aber nicht unbedingt gegeben sein muss.
Mengen an sich ssind erst einmal ungeordnet, und damit nicht durch
Ordinalzahlen erfassbar. Um sie dennoch vergleichen zu koennen, bedarf es
der Kardinalzahlen, sprich der Maechtigkeit.
Ordinalzahlen ("Anzahlen") sind nur auf geordneten Mengen sinnvoll definierbar
(das kam schon in dem Cantor-Zitat vor, dass WM gebracht hat). Fuer Mengen
ohne eine Ordnung hat man nur die Maechtigkeit (sprich die Kardinalzahlen)
als Konzept zum Vergleich.
Post by Me
Post by Juergen Ilse
Bei Mengen ohne weitere Struktur (also insbesondere ohne Ordnung
bzw. Reihenfolge) ist der Begriff der Ordinalzahl sinnlos, dort
ist allein die Maechtigkeit ein sinnvolles Mass fuer einen "Groessen-
vergleich von Mengen".
Mag sein. ANDERERSEITS kann a) jede Menge wohlgeordnet werden (!) und
Ja. Allerdings kann man i.d.R. keine konkrete Wohlordnung angeben (spaetestens
bei Mengen mit einer Maechtigkeit groesser als aleph0), und allein die Existenz
einer Wohlordnung ohne sie konkret angeben zu koennen, ist i.d.R. fuer viele
Beweise etwas unhandlich ...
Cantor hat die Bedeutung einer "Vergleichsmethode", die auch bei Mengen
anwendbar ist, fuer die man keine Ordnung kennt, erkannt und im Gegensatz
zu WM diese Vergleichsmethode bzw. den Begriff der Maechtigkeit *nicht*
als sinnlos abqualifiziert. Ansonsten haette er das Konzept der Maechtigkeit
wohl sofort wieder ueber den Haufen geworfen ...
Post by Me
Post by Juergen Ilse
Post by Me
Cantor hat aber in diesem Zusammenhang (wie WM richtig bemerkt) tatsächlich
von "Anzahl" gesprochen.
Aber nicht fuer (ungeordnete) Mengen, sondern nur fuer Mengen, die
*zusaetzlich* mit irgend einer Art von Ordnung versehen wurden.
Cantor muss erkannt haben, dass, dass im unendlichen "Anzahl" (abgeleitet
aus einer Art der Indizierung) und Maechtigkeit *unterschiedliche* Konzepte
sind, die man zwar bei endlichen Mengen gleichsetzen, bei unendlichen
Mengen streng unterscheiden muss. Sonst haette er nicht *beide* Konzepte
beibehalten und in seinen Arbeiten streng unterschieden. Und er hat erkannt,
dass "Anzahl" (bzw.Ordinalzahl) im Zusammenhang mit Mengen i.a. (also auch
solchen, auf denen man noch keine Ordnung definiert hat) ein sinnloses
Konzept ist (er schreibt ja explizit, dass es von einer Ordnung auf der
Menge abhaengt).
Post by Me
Vielleicht ist es sinnvoll in diesem Zusammenhang zu bemerken, dass Cantor den "Wohlordnungssatz" (also dass jede Menge wohlgeordnet werden kann) für eine "Denknotwendigkeit" hielt.
Cantor muss also erkannt haben, dass die *selbe* Menge mit unterschiedlilchen
Ordnungen zu *unterschiedlichen* Ordinalzahlen fuehren kann, waehrend die
*Kardinalzahl* immer unabhaengig von jeglicher auf der Menge definierten
Ordnung ist. Unds genau das habe ich behauptet. Wenn du es anders verstanden
hast, war es wohl ein Missverstaendnis.

Tschuess,
Juerggen Ilse (***@usenet-verwaltung.de)
Ralf Bader
2020-06-12 17:04:03 UTC
Permalink
Post by Juergen Ilse
Hallo,
Post by Me
Post by Juergen Ilse
Nein. Da er erkannt hat, dass der Begriff "Anzahl" bei unendlichen Mengen
*sinnlos* ist,
Nein, so etwas hat er nicht "erkannt" (jedenfalls -meines Wissens nach- nie zum Ausdruck gebracht).
Doch.
Post by Me
Post by Juergen Ilse
hat er den Begriff der "Maechtigkeit" etabliert, der fuer
endliche Mengen mit "Anzahl der Elemente" uebereinstimmt,
In der Tat.
Post by Juergen Ilse
aber auch bei unendlichen Mengen Sinn ergibt
Ebenso wie der Begriff "Anzahl", der bei Cantor allerdings (anders als bei
Frege) "auf die Anordnung" Bezug nimmt.
Eben. Bei Mengen ohne weitere Struktur (also insbesondere ohne Ordnung
bzw. Reihenfolge) ist der Begriff der Ordinalzahl sinnlos, dort ist allein
die Maechtigkeit ein sinnvolles Mass fuer einen "Groessenvergleich von
Mengen".
Ich hatte Dir schon vor längerer Zeit empfohlen, Dich vor dem Versuch,
schlau zu sein, zu informieren, z.B. anhand eines der zahlreich
verfügbaren Skripten. Im Rahmen des heute üblichen Aufbaus von ZFC ist
das, was Du hier behauptest, Unsinn.
Me
2020-06-12 18:52:28 UTC
Permalink
Post by Ralf Bader
Ich hatte Dir schon vor längerer Zeit empfohlen, Dich vor dem Versuch,
schlau zu sein, zu informieren, z.B. anhand eines der zahlreich
verfügbaren Skripten. Im Rahmen des heute üblichen Aufbaus von ZFC ist
das, was Du hier behauptest, Unsinn.
Ich denke, es ging ihm vor allem darum, darauf Hhinzuweisen, dass man bei unendlichen Mengen statt von der "Anzahl der Elemente" der Menge von der "Kardinalzahl der Menge" sprechen sollte. Tatsächlich kann ich mich daran erinnern, auch schon so etwas gelesen zu haben. Womöglich ist dieser (m. E. bedauerliche) Sachverhalt dem Umstand geschuldet, dass sich Frege und Cantor in Bezug auf diese Frage nicht einig waren. (So weit ich das sehe, konnte sich Cantor mit seiner "Auffassung" nicht durchsetzen; andererseits aber auch Frege nicht mit seiner.)

Grundsätzlich halte ich es jedoch eher mit Frege:

"[...] Wenn man die Bedeutung eines Wortes erweitert, so wird
man darauf zu achten haben, dass möglichst viele allgemeine Sätze ihre
Geltung behalten und zumal so grundlegende, wie für die Anzahl die
Unabhängigkeit von der Reihenfolge ist. Wir haben gar keine Erweiterung
nöthig gehabt, weil unser Begriff der Anzahl sofort auch unendliche Zahlen
umfasst."

Daher sehe ich auch kein Problem darin, von der Anzahl der Elemente der Menge IN zu sprechen. Diese ist dann eben gleich aleph_0.
Ralf Bader
2020-06-12 19:31:40 UTC
Permalink
Post by Me
Post by Ralf Bader
Ich hatte Dir schon vor längerer Zeit empfohlen, Dich vor dem
Versuch, schlau zu sein, zu informieren, z.B. anhand eines der
zahlreich verfügbaren Skripten. Im Rahmen des heute üblichen
Aufbaus von ZFC ist das, was Du hier behauptest, Unsinn.
Ich denke, es ging ihm vor allem darum, darauf Hhinzuweisen, dass man
bei unendlichen Mengen statt von der "Anzahl der Elemente" der Menge
von der "Kardinalzahl der Menge" sprechen sollte. Tatsächlich kann
ich mich daran erinnern, auch schon so etwas gelesen zu haben.
Womöglich ist dieser (m. E. bedauerliche) Sachverhalt dem Umstand
geschuldet, dass sich Frege und Cantor in Bezug auf diese Frage nicht
einig waren. (So weit ich das sehe, konnte sich Cantor mit seiner
"Auffassung" nicht durchsetzen; andererseits aber auch Frege nicht
mit seiner.)
"[...] Wenn man die Bedeutung eines Wortes erweitert, so wird man
darauf zu achten haben, dass möglichst viele allgemeine Sätze ihre
Geltung behalten und zumal so grundlegende, wie für die Anzahl die
Unabhängigkeit von der Reihenfolge ist. Wir haben gar keine
Erweiterung nöthig gehabt, weil unser Begriff der Anzahl sofort auch
unendliche Zahlen umfasst."
Daher sehe ich auch kein Problem darin, von der Anzahl der Elemente
der Menge IN zu sprechen. Diese ist dann eben gleich aleph_0.
Mir ging es eigentlich darum, daß _heute_ im mengentheoretischen
Monismus Ordinalzahlen gewisse Mengen sind, und Kardinalzahlen gewisse
Ordinalzahlen; gegeben eine Menge M, wohlordnet man deren Potenzmenge
und sondert aus der Ordinalzahl dieser Wohlordnung die Menge der zu M
gleichmächtigen Ordinalzahlen aus, und nimmt von diesen wiederum die
kleinste. Das ist die Kardinalzahl von M, und das ist eine Ordinalzahl,
die M kanonisch zugeordnet ist.

Folglich ist die Behauptung "Bei Mengen ohne weitere Struktur (also
insbesondere ohne Ordnung bzw. Reihenfolge) ist der Begriff der
Ordinalzahl sinnlos" Unsinn.
Juergen Ilse
2020-06-12 21:19:45 UTC
Permalink
Hallo,
Post by Me
Post by Ralf Bader
Ich hatte Dir schon vor längerer Zeit empfohlen, Dich vor dem Versuch,
schlau zu sein, zu informieren, z.B. anhand eines der zahlreich
verfügbaren Skripten. Im Rahmen des heute üblichen Aufbaus von ZFC ist
das, was Du hier behauptest, Unsinn.
Ich denke, es ging ihm vor allem darum, darauf Hhinzuweisen, dass man bei unendlichen Mengen statt von der "Anzahl der Elemente" der Menge von der "Kardinalzahl der Menge" sprechen sollte.
Im wesentlichen ja.Vor allemkommt es mir darauf an, dass WMs dusselige
Folgerungen voellig verquer und unsinnig sind, weil er bei unendlichen
Mengen (ohne Beruecksichtigung einer Anordnung der Elemente) von "Anzahl
von Elementen" spricht,obwohl der einzig sinnvolle Vergleich solcher
Mengen eigentlich auf der Maechtigkeit (oder auf einer evt. existierenden
Teilmengenbeziewhung beider Mengen) beruhen kann. WM hat dagegen (wenn ich
mich recht erinnere) in einem kuerzlich hier praesentierten Beitrag ange-
deutet, der Begriff der Maechtigkeit von Mengen sei voellig ueberfluessig.

Wer von den frueheren Mathematikern damals nun die Kardinalitaet und wer die
Onalzahl als "Anzahl" bezeichnete,ist mir voellig Wumpe, da sich der Begriff
"Anzahl" fuer keins von beiden durchgesetzt hat. Ausser WM benutzt meiner
Beobachtung nach heutzutage so gut wie biemand mehr Begriffe wie "Anzahl
der Elemente" fuer unendliche Mengen.

Tschuess,
Juergen Ilse (***@usenet-verwaltung.de)
Ganzhinterseher
2020-06-13 10:24:05 UTC
Permalink
Post by Juergen Ilse
Hallo,
Post by Me
Post by Ralf Bader
Ich hatte Dir schon vor längerer Zeit empfohlen, Dich vor dem Versuch,
schlau zu sein, zu informieren, z.B. anhand eines der zahlreich
verfügbaren Skripten. Im Rahmen des heute üblichen Aufbaus von ZFC ist
das, was Du hier behauptest, Unsinn.
Ich denke, es ging ihm vor allem darum, darauf Hhinzuweisen, dass man bei unendlichen Mengen statt von der "Anzahl der Elemente" der Menge von der "Kardinalzahl der Menge" sprechen sollte.
Im wesentlichen ja.Vor allemkommt es mir darauf an, dass WMs dusselige
Folgerungen voellig verquer und unsinnig sind, weil er bei unendlichen
Mengen (ohne Beruecksichtigung einer Anordnung der Elemente) von "Anzahl
von Elementen" spricht,obwohl der einzig sinnvolle Vergleich solcher
Mengen eigentlich auf der Maechtigkeit (oder auf einer evt. existierenden
Teilmengenbeziewhung beider Mengen) beruhen kann. WM hat dagegen (wenn ich
mich recht erinnere) in einem kuerzlich hier praesentierten Beitrag ange-
deutet, der Begriff der Maechtigkeit von Mengen sei voellig ueberfluessig.
Wer von den frueheren Mathematikern damals nun die Kardinalitaet und wer die
Onalzahl als "Anzahl" bezeichnete,ist mir voellig Wumpe, da sich der Begriff
"Anzahl" fuer keins von beiden durchgesetzt hat. Ausser WM benutzt meiner
Beobachtung nach heutzutage so gut wie biemand mehr Begriffe wie "Anzahl
der Elemente" fuer unendliche Mengen.
Nun höre einmal: Wenn es Bijektionen zwischen unendlichen Mengen gibt, bei denen also jedem Element der Menge A genau ein Element der Menge B zugeordnet wird und umgekehrt, dann haben beide Mengen genau dieselbe Anzahl von Elementen.

Aber wie jeder leicht erkennen kann, besitzt das Interval (0, 1] genau halb so viele rationale und irrationale Elemente wie das Intervall (0, 2]. Das geht aus der Translationsinvarianz hervor, die noch niemand bestritten hat und die auch nicht bestreitbar ist. Die Bijektion versagt also und gibt ein völlig falsches Ergebnis - ob man das Ergebnis nun irgendwie verbrämt und verschleiert zu "verstehen" behauptet oder nicht. Nur wer wer es *nicht* "versteht", sondern erkennt, dass es falsch ist, kann als Mathematiker bezeichnet werden. Alles andere ist Matheologie und außerdem sinnlos, da jede Anwendung dieses Ergebnisses zu Unsinn führt.

Die einzige Erklärung ist die, dass die Bijektion nicht alle Elemente erfasst, sondern nur potentiell unendlich viele. Dann stehen natürlich alle definierbaren Elemente aller Mengen "in Bijektion". (Dabei bezieht sich die Surjektivität natürlich nur auf die potentiell unendlich vielen, nicht auf die aktual unendlich vielen.)

Gruß, WM
Mostowski Collapse
2020-06-13 10:35:23 UTC
Permalink
Die Elemente von Q und R können
nicht bijektiv aufeinander abgebildet werden.
Post by Ganzhinterseher
Post by Juergen Ilse
Hallo,
Post by Me
Post by Ralf Bader
Ich hatte Dir schon vor längerer Zeit empfohlen, Dich vor dem Versuch,
schlau zu sein, zu informieren, z.B. anhand eines der zahlreich
verfügbaren Skripten. Im Rahmen des heute üblichen Aufbaus von ZFC ist
das, was Du hier behauptest, Unsinn.
Ich denke, es ging ihm vor allem darum, darauf Hhinzuweisen, dass man bei unendlichen Mengen statt von der "Anzahl der Elemente" der Menge von der "Kardinalzahl der Menge" sprechen sollte.
Im wesentlichen ja.Vor allemkommt es mir darauf an, dass WMs dusselige
Folgerungen voellig verquer und unsinnig sind, weil er bei unendlichen
Mengen (ohne Beruecksichtigung einer Anordnung der Elemente) von "Anzahl
von Elementen" spricht,obwohl der einzig sinnvolle Vergleich solcher
Mengen eigentlich auf der Maechtigkeit (oder auf einer evt. existierenden
Teilmengenbeziewhung beider Mengen) beruhen kann. WM hat dagegen (wenn ich
mich recht erinnere) in einem kuerzlich hier praesentierten Beitrag ange-
deutet, der Begriff der Maechtigkeit von Mengen sei voellig ueberfluessig.
Wer von den frueheren Mathematikern damals nun die Kardinalitaet und wer die
Onalzahl als "Anzahl" bezeichnete,ist mir voellig Wumpe, da sich der Begriff
"Anzahl" fuer keins von beiden durchgesetzt hat. Ausser WM benutzt meiner
Beobachtung nach heutzutage so gut wie biemand mehr Begriffe wie "Anzahl
der Elemente" fuer unendliche Mengen.
Nun höre einmal: Wenn es Bijektionen zwischen unendlichen Mengen gibt, bei denen also jedem Element der Menge A genau ein Element der Menge B zugeordnet wird und umgekehrt, dann haben beide Mengen genau dieselbe Anzahl von Elementen.
Aber wie jeder leicht erkennen kann, besitzt das Interval (0, 1] genau halb so viele rationale und irrationale Elemente wie das Intervall (0, 2]. Das geht aus der Translationsinvarianz hervor, die noch niemand bestritten hat und die auch nicht bestreitbar ist. Die Bijektion versagt also und gibt ein völlig falsches Ergebnis - ob man das Ergebnis nun irgendwie verbrämt und verschleiert zu "verstehen" behauptet oder nicht. Nur wer wer es *nicht* "versteht", sondern erkennt, dass es falsch ist, kann als Mathematiker bezeichnet werden. Alles andere ist Matheologie und außerdem sinnlos, da jede Anwendung dieses Ergebnisses zu Unsinn führt.
Die einzige Erklärung ist die, dass die Bijektion nicht alle Elemente erfasst, sondern nur potentiell unendlich viele. Dann stehen natürlich alle definierbaren Elemente aller Mengen "in Bijektion". (Dabei bezieht sich die Surjektivität natürlich nur auf die potentiell unendlich vielen, nicht auf die aktual unendlich vielen.)
Gruß, WM
Mostowski Collapse
2020-06-13 10:38:23 UTC
Permalink
Weil R einen höheren Grad an Unendlichkeit aufweist als Q.
Post by Mostowski Collapse
Die Elemente von Q und R können
nicht bijektiv aufeinander abgebildet werden.
Post by Ganzhinterseher
Post by Juergen Ilse
Hallo,
Post by Me
Post by Ralf Bader
Ich hatte Dir schon vor längerer Zeit empfohlen, Dich vor dem Versuch,
schlau zu sein, zu informieren, z.B. anhand eines der zahlreich
verfügbaren Skripten. Im Rahmen des heute üblichen Aufbaus von ZFC ist
das, was Du hier behauptest, Unsinn.
Ich denke, es ging ihm vor allem darum, darauf Hhinzuweisen, dass man bei unendlichen Mengen statt von der "Anzahl der Elemente" der Menge von der "Kardinalzahl der Menge" sprechen sollte.
Im wesentlichen ja.Vor allemkommt es mir darauf an, dass WMs dusselige
Folgerungen voellig verquer und unsinnig sind, weil er bei unendlichen
Mengen (ohne Beruecksichtigung einer Anordnung der Elemente) von "Anzahl
von Elementen" spricht,obwohl der einzig sinnvolle Vergleich solcher
Mengen eigentlich auf der Maechtigkeit (oder auf einer evt. existierenden
Teilmengenbeziewhung beider Mengen) beruhen kann. WM hat dagegen (wenn ich
mich recht erinnere) in einem kuerzlich hier praesentierten Beitrag ange-
deutet, der Begriff der Maechtigkeit von Mengen sei voellig ueberfluessig.
Wer von den frueheren Mathematikern damals nun die Kardinalitaet und wer die
Onalzahl als "Anzahl" bezeichnete,ist mir voellig Wumpe, da sich der Begriff
"Anzahl" fuer keins von beiden durchgesetzt hat. Ausser WM benutzt meiner
Beobachtung nach heutzutage so gut wie biemand mehr Begriffe wie "Anzahl
der Elemente" fuer unendliche Mengen.
Nun höre einmal: Wenn es Bijektionen zwischen unendlichen Mengen gibt, bei denen also jedem Element der Menge A genau ein Element der Menge B zugeordnet wird und umgekehrt, dann haben beide Mengen genau dieselbe Anzahl von Elementen.
Aber wie jeder leicht erkennen kann, besitzt das Interval (0, 1] genau halb so viele rationale und irrationale Elemente wie das Intervall (0, 2]. Das geht aus der Translationsinvarianz hervor, die noch niemand bestritten hat und die auch nicht bestreitbar ist. Die Bijektion versagt also und gibt ein völlig falsches Ergebnis - ob man das Ergebnis nun irgendwie verbrämt und verschleiert zu "verstehen" behauptet oder nicht. Nur wer wer es *nicht* "versteht", sondern erkennt, dass es falsch ist, kann als Mathematiker bezeichnet werden. Alles andere ist Matheologie und außerdem sinnlos, da jede Anwendung dieses Ergebnisses zu Unsinn führt.
Die einzige Erklärung ist die, dass die Bijektion nicht alle Elemente erfasst, sondern nur potentiell unendlich viele. Dann stehen natürlich alle definierbaren Elemente aller Mengen "in Bijektion". (Dabei bezieht sich die Surjektivität natürlich nur auf die potentiell unendlich vielen, nicht auf die aktual unendlich vielen.)
Gruß, WM
Ganzhinterseher
2020-06-13 11:25:46 UTC
Permalink
Post by Mostowski Collapse
Weil R einen höheren Grad an Unendlichkeit aufweist als Q.
Nein. Das Diagonalverfahren setzt eine vollständige Abzählung der Eintragungen voraus, was falsch ist, wenn die rationalen Punkte eines Quarkradius weniger sind als alle rationalen Punkte im Universum.

Überdies setzt es die vollständige Determination eines Grenzwertes durch eine Ziffernfolge voraus, was Humbug ist.

Gruß, WM
jvr
2020-06-13 11:33:40 UTC
Permalink
Post by Ganzhinterseher
Post by Mostowski Collapse
Weil R einen höheren Grad an Unendlichkeit aufweist als Q.
Nein. Das Diagonalverfahren setzt eine vollständige Abzählung der Eintragungen voraus, was falsch ist, wenn die rationalen Punkte eines Quarkradius weniger sind als alle rationalen Punkte im Universum.
Überdies setzt es die vollständige Determination eines Grenzwertes durch eine Ziffernfolge voraus, was Humbug ist.
Gruß, WM
Der Quark, den Sie fabrizieren, hat keinen Radius - die 'quarks' der Physik
übrigens auch nicht - und hätte er einen, so wäre darauf kein einziger
rationaler Punkt.
Ganzhinterseher
2020-06-13 12:29:51 UTC
Permalink
Post by jvr
Post by Ganzhinterseher
Post by Mostowski Collapse
Weil R einen höheren Grad an Unendlichkeit aufweist als Q.
Nein. Das Diagonalverfahren setzt eine vollständige Abzählung der Eintragungen voraus, was falsch ist, wenn die rationalen Punkte eines Quarkradius weniger sind als alle rationalen Punkte im Universum.
Überdies setzt es die vollständige Determination eines Grenzwertes durch eine Ziffernfolge voraus, was Humbug ist.
Der Quark, den Sie fabrizieren, hat keinen Radius - die 'quarks' der Physik
übrigens auch nicht
Du glaubst wohl an die Cantorschen Kraftzentren? "ausdehnungslos, also geometrisch gesprochen für rein punctuell" und sicher auch daran, dass es überabzählbar viele solche "Ätheratome" gibt? Lass Dir gesagt sein: Ein Teilchen, das bisher noch nicht vermessen werden konnte, ist deswegen noch lange nicht ausdehnungslos.
Post by jvr
- und hätte er einen, so wäre darauf kein einziger
rationaler Punkt.
Lass Dich erinnern: 0 ist ein rationaler Punkt.

Gruß, WM
Mostowski Collapse
2020-06-13 16:50:21 UTC
Permalink
Nö, Diagonale hat nicht mit Abzählung zu. Diagonale
kann ich in der Mengenlehre aus dem Aussonderungsaxiom
ableiten. Ist ja nichts anderes als:

d = { x e D | ~(x e f(x)) }

https://de.wikipedia.org/wiki/Aussonderungsaxiom

dann können sie zeigen:

~exists y e D (f(y) = d)

Beweis:
Angenommen f(y)=d. Fall 1: y e d, dann
y e f(y), dann ~(y e d), Widerspruch.
Fall 2: ~(y e d), dann ~(y e f(y)),
dann y e d, Widerspruch.
Also ~(y e d) und (y e d), schon wieder
Widerspruch. Also gibt es kein Urbild
der Diagonale.

Q.E.D.
Post by Ganzhinterseher
Post by Mostowski Collapse
Weil R einen höheren Grad an Unendlichkeit aufweist als Q.
Nein. Das Diagonalverfahren setzt eine vollständige Abzählung der Eintragungen voraus, was falsch ist, wenn die rationalen Punkte eines Quarkradius weniger sind als alle rationalen Punkte im Universum.
Überdies setzt es die vollständige Determination eines Grenzwertes durch eine Ziffernfolge voraus, was Humbug ist.
Gruß, WM
Mostowski Collapse
2020-06-13 16:51:16 UTC
Permalink
Es gibt kein Abzählung, Anzählungs, Zählungs
Axiom in ZFC. Was soll dieses "Counting" sein?
Post by Mostowski Collapse
Nö, Diagonale hat nicht mit Abzählung zu. Diagonale
kann ich in der Mengenlehre aus dem Aussonderungsaxiom
d = { x e D | ~(x e f(x)) }
https://de.wikipedia.org/wiki/Aussonderungsaxiom
~exists y e D (f(y) = d)
Angenommen f(y)=d. Fall 1: y e d, dann
y e f(y), dann ~(y e d), Widerspruch.
Fall 2: ~(y e d), dann ~(y e f(y)),
dann y e d, Widerspruch.
Also ~(y e d) und (y e d), schon wieder
Widerspruch. Also gibt es kein Urbild
der Diagonale.
Q.E.D.
Post by Ganzhinterseher
Post by Mostowski Collapse
Weil R einen höheren Grad an Unendlichkeit aufweist als Q.
Nein. Das Diagonalverfahren setzt eine vollständige Abzählung der Eintragungen voraus, was falsch ist, wenn die rationalen Punkte eines Quarkradius weniger sind als alle rationalen Punkte im Universum.
Überdies setzt es die vollständige Determination eines Grenzwertes durch eine Ziffernfolge voraus, was Humbug ist.
Gruß, WM
Ganzhinterseher
2020-06-13 17:21:10 UTC
Permalink
Post by Mostowski Collapse
Nö, Diagonale hat nicht mit Abzählung zu. Diagonale
kann ich in der Mengenlehre aus dem Aussonderungsaxiom
ableiten.
In der Mathematik existiert eine Diagonale nur in einem fertigen Quadrat mit genau so vielen Zeilen wie Spalten. Und ohne Bildungsgesetz ist eine Diagonalzahl nur dann definiert, wenn auch die letzte Ziffer bekannt ist. Andernfalls wird lediglich ein rationales Intervall definiert.

Gruß, WM
Mostowski Collapse
2020-06-13 18:20:17 UTC
Permalink
Als Bildungsgesetz genügt das Aussonderungsaxiom.
Es gibt kein Abzählung, Anzählungs, Zählungs
Axiom in ZFC. Was soll dieses "Counting" sein?

Diagonale kann ich in der Mengenlehre aus dem
Aussonderungsaxiom ableiten. Ist ja nichts anderes als:

d = { x e D | ~(x e f(x)) }

https://de.wikipedia.org/wiki/Aussonderungsaxiom

dann können Sie zeigen:

~exists y e D (f(y) = d)

Beweis:
Angenommen f(y)=d. Fall 1: y e d, dann
y e f(y), dann ~(y e d), Widerspruch.
Fall 2: ~(y e d), dann ~(y e f(y)),
dann y e d, Widerspruch.
Also ~(y e d) und (y e d), schon wieder
Widerspruch. Also gibt es kein Urbild
der Diagonale.

Q.E.D.
Post by Ganzhinterseher
Post by Mostowski Collapse
Nö, Diagonale hat nicht mit Abzählung zu. Diagonale
kann ich in der Mengenlehre aus dem Aussonderungsaxiom
ableiten.
In der Mathematik existiert eine Diagonale nur in einem fertigen Quadrat mit genau so vielen Zeilen wie Spalten. Und ohne Bildungsgesetz ist eine Diagonalzahl nur dann definiert, wenn auch die letzte Ziffer bekannt ist. Andernfalls wird lediglich ein rationales Intervall definiert.
Gruß, WM
Mostowski Collapse
2020-06-13 18:33:23 UTC
Permalink
Man könnte vielleicht MathRealism so interpretieren,
dass es das Aussonderungsaxiom verweigert, und
an dessen Stelle ein Countingaxiom setzt.

Vielleicht würde das auch Mückenschuss, Mücken-
verdruss und Mückenpanne erklären. Vielleicht
sogar die Etymologie von Mückenheim.
Post by Mostowski Collapse
Als Bildungsgesetz genügt das Aussonderungsaxiom.
Es gibt kein Abzählung, Anzählungs, Zählungs
Axiom in ZFC. Was soll dieses "Counting" sein?
Diagonale kann ich in der Mengenlehre aus dem
d = { x e D | ~(x e f(x)) }
https://de.wikipedia.org/wiki/Aussonderungsaxiom
~exists y e D (f(y) = d)
Angenommen f(y)=d. Fall 1: y e d, dann
y e f(y), dann ~(y e d), Widerspruch.
Fall 2: ~(y e d), dann ~(y e f(y)),
dann y e d, Widerspruch.
Also ~(y e d) und (y e d), schon wieder
Widerspruch. Also gibt es kein Urbild
der Diagonale.
Q.E.D.
Post by Ganzhinterseher
Post by Mostowski Collapse
Nö, Diagonale hat nicht mit Abzählung zu. Diagonale
kann ich in der Mengenlehre aus dem Aussonderungsaxiom
ableiten.
In der Mathematik existiert eine Diagonale nur in einem fertigen Quadrat mit genau so vielen Zeilen wie Spalten. Und ohne Bildungsgesetz ist eine Diagonalzahl nur dann definiert, wenn auch die letzte Ziffer bekannt ist. Andernfalls wird lediglich ein rationales Intervall definiert.
Gruß, WM
Ganzhinterseher
2020-06-13 11:23:15 UTC
Permalink
Post by Mostowski Collapse
Die Elemente von Q und R können
nicht bijektiv aufeinander abgebildet werden.
Die Definierbaren schon.

Wäre Königs Satz, daß alle „endlich definirbaren" reellen Zahlen einen Inbegriff von der Mächtigkeit alef_0 ausmachen, richtig, so hieße dies, das ganze Zahlencontinuum sei abzählbar, was doch sicherlich falsch ist. (Cantor)

Nein, das ist mit Sicherheit richtig.

Gruß, WM
Mostowski Collapse
2020-06-13 16:42:24 UTC
Permalink
Dann ist es aber keine Bijektion zwischen
Q und R mehr. R beinhaltet alle Elemente,
nicht nur eine abzählbare Untermenge.
Post by Ganzhinterseher
Post by Mostowski Collapse
Die Elemente von Q und R können
nicht bijektiv aufeinander abgebildet werden.
Die Definierbaren schon.
Wäre Königs Satz, daß alle „endlich definirbaren" reellen Zahlen einen Inbegriff von der Mächtigkeit alef_0 ausmachen, richtig, so hieße dies, das ganze Zahlencontinuum sei abzählbar, was doch sicherlich falsch ist. (Cantor)
Nein, das ist mit Sicherheit richtig.
Gruß, WM
Ganzhinterseher
2020-06-13 17:15:45 UTC
Permalink
Post by Mostowski Collapse
Dann ist es aber keine Bijektion zwischen
Q und R mehr. R beinhaltet alle Elemente,
nicht nur eine abzählbare Untermenge.
„Unendliche Definitionen" (die nicht in endlicher Zeit verlaufen) sind Undinge. (Cantor)

Darunter fallen auch die nur durch unendliche Ziffernfolgen "definierten" Elemente.

Gruß, WM
Juergen Ilse
2020-06-13 11:23:25 UTC
Permalink
Hallo,
Post by Ganzhinterseher
Post by Juergen Ilse
Post by Me
Post by Ralf Bader
Ich hatte Dir schon vor längerer Zeit empfohlen, Dich vor dem Versuch,
schlau zu sein, zu informieren, z.B. anhand eines der zahlreich
verfügbaren Skripten. Im Rahmen des heute üblichen Aufbaus von ZFC ist
das, was Du hier behauptest, Unsinn.
Ich denke, es ging ihm vor allem darum, darauf Hhinzuweisen, dass man bei unendlichen Mengen statt von der "Anzahl der Elemente" der Menge von der "Kardinalzahl der Menge" sprechen sollte.
Im wesentlichen ja.Vor allemkommt es mir darauf an, dass WMs dusselige
Folgerungen voellig verquer und unsinnig sind, weil er bei unendlichen
Mengen (ohne Beruecksichtigung einer Anordnung der Elemente) von "Anzahl
von Elementen" spricht,obwohl der einzig sinnvolle Vergleich solcher
Mengen eigentlich auf der Maechtigkeit (oder auf einer evt. existierenden
Teilmengenbeziewhung beider Mengen) beruhen kann. WM hat dagegen (wenn ich
mich recht erinnere) in einem kuerzlich hier praesentierten Beitrag ange-
deutet, der Begriff der Maechtigkeit von Mengen sei voellig ueberfluessig.
Wer von den frueheren Mathematikern damals nun die Kardinalitaet und wer die
Onalzahl als "Anzahl" bezeichnete,ist mir voellig Wumpe, da sich der Begriff
"Anzahl" fuer keins von beiden durchgesetzt hat. Ausser WM benutzt meiner
Beobachtung nach heutzutage so gut wie biemand mehr Begriffe wie "Anzahl
der Elemente" fuer unendliche Mengen.
Nun höre einmal: Wenn es Bijektionen zwischen unendlichen Mengen gibt, bei denen also jedem Element der Menge A genau ein Element der Menge B zugeordnet wird und umgekehrt, dann haben beide Mengen genau dieselbe Anzahl von Elementen.
In der Formulierung der heutigen Mathematik heisst das "beide Mengen haben
die selbe Maechtigkeit".
Post by Ganzhinterseher
Aber wie jeder leicht erkennen kann, besitzt das Interval (0, 1] genau halb so viele rationale und irrationale Elemente wie das Intervall (0, 2].
Fuer das rechnen mit Kardinalzahlen gilt: 2 * aleph0 = aleph0 , auch wenn
IHNEN das nicht einleuchtet.
Post by Ganzhinterseher
Das geht aus der Translationsinvarianz hervor, die noch niemand bestritten hat und die auch nicht bestreitbar ist.
SIE sind also immer noch zu daemlich, um "dedekind-Unendlichkeit" zu begreifen.

Tschuess,
Juergen Ilse (***@usenet-verwaltung.de)
Me
2020-06-14 00:10:49 UTC
Permalink
Aber wie jeder leicht erkennen kann, besitzt das Intervall (0, 1] genau halb
so viele rationale und irrationale Elemente wie das Intervall (0, 2].
Nein, Du Dummschwätzer, das kann nicht jeder "leicht erkennen". Denn man kann die beiden Intervalle auch als /gleichzahlig/ auffassen, da es eine Bijektion von (0, 1] auf (0, 2] gibt: f(x) = 2x (für alle x e (0, 1]).

Hinweis: "Ich will der Kürze wegen den Begriff F dem Begriffe G gleichzahlig nennen, wenn die Möglichkeit vorliegt [die unter den einen den unter den andern Begriff fallenden Gegenständen beiderseits eindeutig zuzuordnen], muss aber bitten, dies Wort als eine willkührlich gewählte Bezeichnungsweise zu betrachten, deren Bedeutung nicht der sprachlichen Zusammensetzung, sondern dieser Festsetzung zu entnehmen ist."

"Es ist nöthig, die Gleichzahligkeit noch etwas genauer zu fassen. Wir erklärten sie mittels der beiderseits eindeutigen Zuordnung, und wie ich diesen Ausdruck verstehen will, ist jetzt darzulegen, weil man leicht etwas Anschauliches darin vermuthen könnte.
Betrachten wir folgendes Beispiel! Wenn ein Kellner sicher sein will, dass er ebensoviele Messer als Teller auf den Tisch legt, braucht er weder diese noch jene zu zählen, wenn er nur rechts neben jeden Teller ein Messer legt, sodass jedes Messer auf dem Tische sich rechts neben einem Teller befindet. Die Teller und Messer sind so beiderseits eindeutig einander zugeordnet ..."

Wenn Du zu blöde bist, das zu begreifen, kann man natürlich nichts machen. Cantor sprach in diesem Zusammenhang auch von der "Gleichmächtigkeit" der Mengen.

In jedem Fall haben beide Mengen die gleiche Kardinalzahl.
WM
2020-06-14 15:45:53 UTC
Permalink
Post by Me
Aber wie jeder leicht erkennen kann, besitzt das Intervall (0, 1] genau halb
so viele rationale und irrationale Elemente wie das Intervall (0, 2].
das kann nicht jeder "leicht erkennen".
Doch, dafür gibt es einen einfachen und schlagenden Beweis: Alle Zahlen der Form 0,xyz... gibt es auch in der Version 1,xyz...
Post by Me
Denn man kann die beiden Intervalle auch als /gleichzahlig/ auffassen, da es eine Bijektion von (0, 1] auf (0, 2] gibt: f(x) = 2x (für alle x e (0, 1]).
Das zeigt unmittelbar, dass die Intervalle nicht beide ausgeschöpft werden. Kein Intervall wird ausgeschöpft. Beweis: Für alle Indizes der ausgeschöpften rationalen Zahlen gilt ω| - n = |ω|.
Post by Me
Hinweis: "Ich will der Kürze wegen den Begriff F dem Begriffe G gleichzahlig nennen, wenn die Möglichkeit vorliegt [die unter den einen den unter den andern Begriff fallenden Gegenständen beiderseits eindeutig zuzuordnen], muss aber bitten, dies Wort als eine willkührlich gewählte Bezeichnungsweise zu betrachten, deren Bedeutung nicht der sprachlichen Zusammensetzung, sondern dieser Festsetzung zu entnehmen ist."
"Es ist nöthig, die Gleichzahligkeit noch etwas genauer zu fassen. Wir erklärten sie mittels der beiderseits eindeutigen Zuordnung, und wie ich diesen Ausdruck verstehen will, ist jetzt darzulegen, weil man leicht etwas Anschauliches darin vermuthen könnte.
Betrachten wir folgendes Beispiel! Wenn ein Kellner sicher sein will, dass er ebensoviele Messer als Teller auf den Tisch legt, braucht er weder diese noch jene zu zählen, wenn er nur rechts neben jeden Teller ein Messer legt, sodass jedes Messer auf dem Tische sich rechts neben einem Teller befindet. Die Teller und Messer sind so beiderseits eindeutig einander zugeordnet ..."
Wenn Du zu blöde bist, das zu begreifen, kann man natürlich nichts machen.
Zu begreifen bleibt hier allein Folgendes: Für endliche Mengen ist die Bijektion ein zutreffendes Maß. Für unendliche Mengen versagt dieses Maß.

Du bist doch sonst überall mit dem Argument zur Stelle, dass für unendliche Mengen eben andere Gesetze gelten als für endliche. Weshalb versagt Dein Verstand hier?
Post by Me
Cantor sprach in diesem Zusammenhang auch von der "Gleichmächtigkeit" der Mengen.
In jedem Fall haben beide Mengen die gleiche Kardinalzahl.
Das mag sein, aber diese Zahl hat keinerlei mathematische Bedeutung. Die Menge der rationalen Punkte auf der reellen Achse hat jedenfalls nichts mit der anzählbaren Menge der gekürzten Brüche zu tun. Das zeigt sich an der mathematischen Grundwahrheit, dass zwischen einem irrationalen Punkt und einem rein irrationalen Intervall auf der reellen Achse ein rationaler Punkt liegen muss. Deswegen gibt es übrigens kein rein irrationales Intervall, und deshalb ist das Maß der reellen Achse eine Widerlegung des Cantorschen Abzählbarkeitsbegriffs.

Gruß, WM
Me
2020-06-14 18:47:25 UTC
Permalink
Post by WM
Post by Me
Cantor sprach in diesem Zusammenhang auch von der "Gleichmächtigkeit" der Mengen.
In jedem Fall haben beide Mengen die gleiche Kardinalzahl.
Das mag sein, aber diese Zahl hat keinerlei mathematische Bedeutung.
Ach, Kardinalzahlen haben keinerlei Mathematische Bedeutung? Oder geht es Ihnen nur um die Kardinalzahl des "Kontinuums"?

Nun, das ist natürlich wieder mal eine ausgesprochen dumme/unsinnige Behauptung Ihrerseits.

Jedenfalls SO VIEL mathematisches Verständnis sollte man aufbringen, zu verstehen, dass ein mengentheoretisch geführter Beweis für die Tatsache, dass es UNENDLICH viele (ja sogar überabzählbar viele) transzendente Zahlen gibt, durchaus eine "mathematische Bedeutung" hat.

Sie sind einfach für jegliche Art der Mathematik zu blöde, Herr Mückenheim.

Für alle anderen: https://de.wikipedia.org/wiki/Transzendente_Zahl#Beweis_der_%C3%9Cberabz%C3%A4hlbarkeit_durch_Georg_Cantor
Ganzhinterseher
2020-06-15 14:47:37 UTC
Permalink
Post by Me
Post by WM
Post by Me
Cantor sprach in diesem Zusammenhang auch von der "Gleichmächtigkeit" der Mengen.
In jedem Fall haben beide Mengen die gleiche Kardinalzahl.
Das mag sein, aber diese Zahl hat keinerlei mathematische Bedeutung.
Ach, Kardinalzahlen haben keinerlei Mathematische Bedeutung? Oder geht es Ihnen nur um die Kardinalzahl des "Kontinuums"?
Kardinalzahlen allgemein haben keine mathematische Bedeutung. Die Kardinalzahl ℵo zeigt nur an, dass eine Menge unendlich ist und ihre definierbaren Elemente mit den definierbaren natürlichen Zahlen gepaart werden können.

Die Bijektion mit allen natürlichen Zahlen ist nicht möglich und führt außerdem zu einem Widerspruch, denn Bijektionen sind sehr genau, besser als auf plusminus ein Element. Im Falle unendlicher Mengen versagt diese Genauigkeit, denn für die Menge der Primzahlen ergibt sich genau dieselbe Bijektion mit |N wie für die Menge der Brüche. Wie zu erwarten geht die Bijektion also schief.
Post by Me
Nun, das ist natürlich wieder mal eine ausgesprochen dumme/unsinnige Behauptung Ihrerseits.
Im Gegenteil, die Bijektion als Maß für unendliche Mengen ist leicht erkennbar falsch.
Post by Me
Jedenfalls SO VIEL mathematisches Verständnis sollte man aufbringen, zu verstehen, dass ein mengentheoretisch geführter Beweis für die Tatsache, dass es UNENDLICH viele (ja sogar überabzählbar viele) transzendente Zahlen gibt, durchaus eine "mathematische Bedeutung" hat.
Dieser Beweis setzt die Möglichkeit der Vollendung unendlicher Listen voraus, was, wie wir im Falle der Bijektion mit |N gesehen haben, nicht funktioniert. Deswegen ist er irrelevant.

Hast Du Dir wirklich noch nie überlegt, dass eine Bijektion als eins-zu-eins- Abbildung tatsächlich außerordentlich präzise sein müsste? Die Abzählbarkeit der Mengen |N, Q und anderer ist doch ein klarer Beweis dafür, dass bei allen diesen Abbildungen dieselbe Anzahl zustandekommt, dass es also nur um eine Anzählbarkeit gehen kann.

Gruß, WM
Me
2020-06-15 15:23:30 UTC
Permalink
On Monday, June 15, 2020 at 4:47:38 PM UTC+2, Ganzhinterseher wrote: [...]

So schreiben dermaßen viel unsinniges Zeug, dass es einem schwer fällt, überhaupt IRGEND ETWAS zu finden, das -wenn auch falsch- wenigstens sinnvoll genug ist, um darauf eine Antworten geben zu können.
für die Menge der Primzahlen ergibt sich genau dieselbe Bijektion mit IN
wie für die Menge der Brüche
Äh, nö. Wenn wir IN in beiden Fällen als "Definitionsmenge" heranziehen, ist die Bildmenge einmal P (die Menge der Primzahlen) und einmal B (die Menge der Brüche). Da klarerweise P =/= B ist, sind natürlich auch je zwei Bijektionen

f: IN --> P
und
g: IN --> B

VERSCHIEDEN. Es gilt also für alle solche Bijektionen f, g: f =/= g.

Man kann konstatieren, dass Sie zumeist entweder sinnloses Zeug von sich geben, oder aber falsche Behauptungen aufstellen), die so offensichtlich falsch sind, dass es beinahe schon lächerlich ist.
Die Abzählbarkeit der Mengen IN, Q und anderer ist doch ein klarer Beweis
dafür, dass bei allen diesen Abbildungen dieselbe Anzahl zustandekommt.
Ja, genau so hat es auch Frege gesehen. :-)

"Indem ich Indem ich hierin, wie ich glaube, mit Cantor übereinstimme, weiche ich doch
in der Benennung etwas von ihm ab. Meine Anzahl nennt er "Mächtigkeit,"
während sein Begriff der Anzahl auf die Anordnung Bezug nimmt. Für endliche
Anzahlen ergiebt sich freilich doch eine Unabhängigkeit von der
Reihenfolge, dagegen nicht für unendlichgrosse. Nun enthält der
Sprachgebrauch des Wortes "Anzahl" und der Frage "wieviele?" keine
Hinweisung auf eine bestimmte Anordnung. Cantors Anzahl antwortet vielmehr
auf die Frage: "das wievielste Glied in der Succession ist das Endglied?"
Darum scheint mir meine Benennung besser mit dem Sprachgebrauche
übereinzustimmen. Wenn man die Bedeutung eines Wortes erweitert, so wird
man darauf zu achten haben, dass möglichst viele allgemeine Sätze ihre
Geltung behalten und zumal so grundlegende, wie für die Anzahl die
Unabhängigkeit von der Reihenfolge ist. Wir haben gar keine Erweiterung
nöthig gehabt, weil unser Begriff der Anzahl sofort auch unendliche Zahlen
umfasst."

(Gottlob Frege, Grundlagen der Arithmetik, 1884)

Und ja, Bijektionen sind das Mittel der Wahl, um "Gleichzahligkeit" (Cantor: Äquivalenz, Gleichmächtigkeit) festzustellen. :-)

"Ich will der Kürze wegen den Begriff F dem Begriffe G /gleichzahlig/ nennen, wenn die Möglichkeit vorliegt [die unter den einen den unter den andern Begriff fallenden Gegenständen beiderseits eindeutig zuzuordnen], muss aber bitten, dies Wort als eine willkührlich gewählte Bezeichnungsweise zu betrachten, deren Bedeutung nicht der sprachlichen Zusammensetzung, sondern dieser Festsetzung zu entnehmen ist." (G. Frege)
Ganzhinterseher
2020-06-15 16:37:04 UTC
Permalink
Post by Me
Die Abzählbarkeit der Mengen IN, Q und anderer ist doch ein klarer Beweis
dafür, dass bei allen diesen Abbildungen dieselbe Anzahl zustandekommt.
Ja, genau so hat es auch Frege gesehen. :-)
Aber er hat nicht den wahren Grund erkannt. Der liegt nämlich nicht darin, dass es genau so viele Brüche wie natürliche Zahlen gäbe. Diese Annahme kann nur durch eine kollektive Psychose ausgelöst worden sein. Die Ursache liegt einfach darin, dass jede "Bijektion" unendlicher Mengen nur die potentiell unendliche Kollektion definierbarer Elemente umfasst.

Gruß, WM
Mostowski Collapse
2020-06-15 17:39:42 UTC
Permalink
Q und N haben den gleichen Grad an Unendlichkeit.
Q und R aber nicht, R ist mächtiger als Q.

Q, N und R erfüllen alle das Dedekind Kriterium
der Unendlichkeit. Was aber noch nichts über

den Grad der Unendlichkeit aussagt.
Post by Ganzhinterseher
Post by Me
Die Abzählbarkeit der Mengen IN, Q und anderer ist doch ein klarer Beweis
dafür, dass bei allen diesen Abbildungen dieselbe Anzahl zustandekommt.
Ja, genau so hat es auch Frege gesehen. :-)
Aber er hat nicht den wahren Grund erkannt. Der liegt nämlich nicht darin, dass es genau so viele Brüche wie natürliche Zahlen gäbe. Diese Annahme kann nur durch eine kollektive Psychose ausgelöst worden sein. Die Ursache liegt einfach darin, dass jede "Bijektion" unendlicher Mengen nur die potentiell unendliche Kollektion definierbarer Elemente umfasst.
Gruß, WM
Uwe Weiss
2020-06-09 17:27:48 UTC
Permalink
Post by Ganzhinterseher
Post by Juergen Ilse
Hallo,
Post by Ganzhinterseher
Dafür gibt es das Wort "Grenzwert", dessen Bedeutung ganz klar definiert ist.
Dann nennen SIE doch vielleicht zur Abwechselung mal eine mathematisch
korrekte Definition, z.B. eine matheatisch korrekte Definition des Begriffs
"Grenzwert".
Der Grenzwert oder Limes einer Folge von Zahlen ist eine Zahl, der die Folge beliebig nah kommt. (Wikipedia)
lim k --> oo [
|{x ∈ ℝ | 0 < x =< 1} ∩ {q_1, q_2, ..., q_k}|
/
|{x ∈ ℝ | 1000 < x =< 1001} ∩ {q_1, q_2, ..., q_k}|]
Ja, ja. Die guten, alten, rationalen Zahlen - in Profi-Kreisen auch
gerne mal mit "ℝ" bezeichnet.

SCNR

-Uwe-
Me
2020-06-09 17:33:15 UTC
Permalink
Post by Uwe Weiss
Post by Ganzhinterseher
lim k --> oo [
|{x ∈ ℝ | 0 < x =< 1} ∩ {q_1, q_2, ..., q_k}|
/
|{x ∈ ℝ | 1000 < x =< 1001} ∩ {q_1, q_2, ..., q_k}|]
Ja, ja. Die guten, alten, rationalen Zahlen - in Profi-Kreisen auch
gerne mal mit "ℝ" bezeichnet.
Das "ℝ" passt hier schon, weil dann jeweils ein Schnitt mit {q_1, q_2, ..., q_k} gebildet wird, die q_i sind rationale Zahlen.
Me
2020-06-07 23:10:27 UTC
Permalink
Post by Ganzhinterseher
Gesucht ist
lim k --> oo [
|{x ∈ ℝ | 0 < x =< 1} ∩ {q_1, q_2, ..., q_k}|
/
|{x ∈ ℝ | 1000 < x =< 1001} ∩ {q_1, q_2, ..., q_k}|]
wobei die q_i der Cantorschen Folge
1/1, 1/2, 2/1, 1/3, 3/1, 1/4, 2/3, 3/2, 4/1, 1/5, 5/1, 1/6, ...
entnommen sind.
Also ich habe das jetzt nicht im Detail analysiert, aber es sieht für mich so aus (=Vermutung), dass lim k --> oo (...) = 0 ist.

So what?

Wichtig ist bei einer Abzählung von Q+ NICHT, dass der Limes irgendwelcher "Verhältnisse" =/= 0 ist, sonder lediglich, dass es eine Bijektion zwischen IN und Q+ _gibt_.

Man kann z. B. die beiden folgenden Mengenfolgen betrachten:

(A_n) mit A_n = {10^1, 10^2, 10^3, ..., 10^n}

und

(B_n) mit B_n = IN \ A_n .

Dann gilt z. B.

lim_(k->oo) (|A_k| / |B_k|) = 0 .

Dennoch gilt card({10^1, 10^2, 10^3, ...}) = aleph_0 und card(IN \ {10^1, 10^2, 10^3, ...}) = aleph_0. Dass der lim = 0 ist, ändert also nichts am Umstand, dass die "Anzahl" der Zahlen in {10^1, 10^2, 10^3, ...} gleich der "Anzahl" der Zahlen in IN \ {10^1, 10^2, 10^3, ...} ist.

Sie sehen mal wieder "Probleme", wo keine sind.
Mostowski Collapse
2020-06-08 13:08:50 UTC
Permalink
"Me" wrote: Sie sehen mal wieder "Probleme", wo keine sind.

I guess WM is expert in Mückendope.
Post by Me
Post by Ganzhinterseher
Gesucht ist
lim k --> oo [
|{x ∈ ℝ | 0 < x =< 1} ∩ {q_1, q_2, ..., q_k}|
/
|{x ∈ ℝ | 1000 < x =< 1001} ∩ {q_1, q_2, ..., q_k}|]
wobei die q_i der Cantorschen Folge
1/1, 1/2, 2/1, 1/3, 3/1, 1/4, 2/3, 3/2, 4/1, 1/5, 5/1, 1/6, ...
entnommen sind.
Also ich habe das jetzt nicht im Detail analysiert, aber es sieht für mich so aus (=Vermutung), dass lim k --> oo (...) = 0 ist.
So what?
Wichtig ist bei einer Abzählung von Q+ NICHT, dass der Limes irgendwelcher "Verhältnisse" =/= 0 ist, sonder lediglich, dass es eine Bijektion zwischen IN und Q+ _gibt_.
(A_n) mit A_n = {10^1, 10^2, 10^3, ..., 10^n}
und
(B_n) mit B_n = IN \ A_n .
Dann gilt z. B.
lim_(k->oo) (|A_k| / |B_k|) = 0 .
Dennoch gilt card({10^1, 10^2, 10^3, ...}) = aleph_0 und card(IN \ {10^1, 10^2, 10^3, ...}) = aleph_0. Dass der lim = 0 ist, ändert also nichts am Umstand, dass die "Anzahl" der Zahlen in {10^1, 10^2, 10^3, ...} gleich der "Anzahl" der Zahlen in IN \ {10^1, 10^2, 10^3, ...} ist.
Sie sehen mal wieder "Probleme", wo keine sind.
Ganzhinterseher
2020-06-08 14:02:20 UTC
Permalink
Post by Me
Post by Ganzhinterseher
Gesucht ist
lim k --> oo [
|{x ∈ ℝ | 0 < x =< 1} ∩ {q_1, q_2, ..., q_k}|
/
|{x ∈ ℝ | 1000 < x =< 1001} ∩ {q_1, q_2, ..., q_k}|]
wobei die q_i der Cantorschen Folge
1/1, 1/2, 2/1, 1/3, 3/1, 1/4, 2/3, 3/2, 4/1, 1/5, 5/1, 1/6, ...
entnommen sind.
Also ich habe das jetzt nicht im Detail analysiert, aber es sieht für mich so aus (=Vermutung), dass lim k --> oo (...) = 0 ist.
Nein, das ist sicher falsch. Vielleicht kommt ja doch noch eine korrekte Lösung. Aber meine Vermutung, > 100, reicht aus, um zu beweisen, dass Cantor ganz gewiss nicht alle erwischt. (https://groups.google.com/forum/#!topic/de.sci.mathematik/00wT38bhx0g%5B1-25%5D)
Post by Me
So what?
Wichtig ist bei einer Abzählung von Q+ NICHT, dass der Limes irgendwelcher "Verhältnisse" =/= 0 ist, sonder lediglich, dass es eine Bijektion zwischen IN und Q+ _gibt_.
Die gibt es aber nicht, denn in allen Intervallen befinden sich genau dieselben Brüche, nur mit n addiert.
Post by Me
(A_n) mit A_n = {10^1, 10^2, 10^3, ..., 10^n}
und
(B_n) mit B_n = IN \ A_n .
Dann gilt z. B.
lim_(k->oo) (|A_k| / |B_k|) = 0 .
Dennoch gilt card({10^1, 10^2, 10^3, ...}) = aleph_0 und card(IN \ {10^1, 10^2, 10^3, ...}) = aleph_0. Dass der lim = 0 ist, ändert also nichts am Umstand, dass die "Anzahl" der Zahlen in {10^1, 10^2, 10^3, ...} gleich der "Anzahl" der Zahlen in IN \ {10^1, 10^2, 10^3, ...} ist.
Man kann nicht einen Unsinn mit einem anderen begründen - jedenfalls nicht, wenn man auch außerhalb einer Gruppe von Narren, die daran glauben und unfähig sind, ihn zu durchschauen, glaubwürdig bleiben will.

Nochmal zur Verdeutlichung: Für alle definierbaren Zahlen ...

∀k ∈ ℕ_def: |ℕ \ {1, 2, 3, ..., k}| = ℵo.

Man kann alle definierbaren Zahlen, also alle, die diese Gleichung erfüllen, in einer Kollektion zusammenfassen. Dann gilt:

ℕ \ ℕ_def = ℵo.

Oder kannst Du auch nur eine einzige definierbare Zahl angeben, die dort fehlen würde?

Gruß, WM
Ralf Bader
2020-06-08 19:42:10 UTC
Permalink
Post by Ganzhinterseher
On Sunday, June 7, 2020 at 6:01:14 PM UTC+2, Ganzhinterseher
Post by Ganzhinterseher
Gesucht ist
lim k --> oo [ |{x ∈ ℝ | 0 < x =< 1} ∩ {q_1, q_2, ..., q_k}| /
|{x ∈ ℝ | 1000 < x =< 1001} ∩ {q_1, q_2, ..., q_k}|]
wobei die q_i der Cantorschen Folge
1/1, 1/2, 2/1, 1/3, 3/1, 1/4, 2/3, 3/2, 4/1, 1/5, 5/1, 1/6, ...
entnommen sind.
Also ich habe das jetzt nicht im Detail analysiert, aber es sieht
für mich so aus (=Vermutung), dass lim k --> oo (...) = 0 ist.
Nein, das ist sicher falsch. Vielleicht kommt ja doch noch eine
korrekte Lösung. Aber meine Vermutung, > 100, reicht aus, um zu
beweisen, dass Cantor ganz gewiss nicht alle erwischt.
(https://groups.google.com/forum/#!topic/de.sci.mathematik/00wT38bhx0g%5B1-25%5D)
So what?
Post by Ganzhinterseher
Wichtig ist bei einer Abzählung von Q+ NICHT, dass der Limes
irgendwelcher "Verhältnisse" =/= 0 ist, sonder lediglich, dass es
eine Bijektion zwischen IN und Q+ _gibt_.
Die gibt es aber nicht, denn in allen Intervallen befinden sich genau
dieselben Brüche, nur mit n addiert.
(A_n) mit A_n = {10^1, 10^2, 10^3, ..., 10^n}
und
(B_n) mit B_n = IN \ A_n .
Dann gilt z. B.
lim_(k->oo) (|A_k| / |B_k|) = 0 .
Dennoch gilt card({10^1, 10^2, 10^3, ...}) = aleph_0 und card(IN \
{10^1, 10^2, 10^3, ...}) = aleph_0. Dass der lim = 0 ist, ändert
also nichts am Umstand, dass die "Anzahl" der Zahlen in {10^1,
10^2, 10^3, ...} gleich der "Anzahl" der Zahlen in IN \ {10^1,
10^2, 10^3, ...} ist.
Man kann nicht einen Unsinn mit einem anderen begründen - jedenfalls
nicht, wenn man auch außerhalb einer Gruppe von Narren, die daran
glauben und unfähig sind, ihn zu durchschauen, glaubwürdig bleiben
will.
Wenn es nicht zwingende Indizien für Gegenteiliges gäbe, dann würde man
die von Ihnen zur Schau gestellte Kombination aus Impertinenz und
Brachialblödheit allenfalls im Rahmen psychologischer Tests daraufhin,
wann den Versuchspersonen Geduldsfaden und Kragen platzen, für möglich
halten.
WM
2020-06-09 16:02:00 UTC
Permalink
Post by Ralf Bader
Wenn es nicht zwingende Indizien für Gegenteiliges gäbe, dann würde man
Deine fgraundlegenden mathematischen Aussagen sind doch immer wieder amüsant zu lesen. Du kannst aber nicht anders?

Gruß, WM
Me
2020-06-09 01:09:36 UTC
Permalink
Post by Me
(A_n) mit A_n = {10^1, 10^2, 10^3, ..., 10^n}
und
(B_n) mit B_n = IN \ A_n .
Gemeint war hier natürlich:

B_n = {m e IN : m <= 10^n} \ A_n .

Dann ist |A_k| = k und |B_k| = 10^k - k für alle k e IN.
Post by Me
Dann gilt z. B.
lim_(k->oo) (|A_k| / |B_k|) = 0 .
Siehe: https://www.wolframalpha.com/input/?i=lim%28k%2F%2810%5Ek+-+k%29%29%2C+k-%3Eoo
Uwe Weiss
2020-06-09 17:22:33 UTC
Permalink
Post by Ganzhinterseher
1/1, 1/2, 2/1, 1/3, 3/1, 1/4, 2/3, 3/2, 4/1, 1/5, 5/1, 1/6, ... .
Bisher hat niemand den Grenzwert des Verhältnisses der Anzahl der positiven rationalen Zahlen aus dem ersten und dem n-ten Einheitsintervall, also aus (0, 1] und (n, n+1], ausrechnen können, nicht einmal das Verhältnis aus (0, 1] und (1000, 1001]. Aber das ist sicher größer als 100.
Mir ist der Zusammenhang zwischen dieser Frage, und "Cantors berühmter
Folge" nicht klar. Was hat das eine mit dem anderen zu tun?

Die Kardinalitäten innerhalb der rationalen Zahlen von (0, 1] und (1000,
1001] sind identisch.

Und ebenso - und DAS ist beängstigend! - die von (0, 1] und (0, 2].
Post by Ganzhinterseher
Kann wenigstens jemand die Bedeutung dieses Verhältnisses interpretieren?
Ja. Wie oben geschrieben: beängstigend!
Post by Ganzhinterseher
Gruß, WM
Ganzhinterseher
2020-06-09 17:38:54 UTC
Permalink
Post by Uwe Weiss
Post by Ganzhinterseher
1/1, 1/2, 2/1, 1/3, 3/1, 1/4, 2/3, 3/2, 4/1, 1/5, 5/1, 1/6, ... .
Bisher hat niemand den Grenzwert des Verhältnisses der Anzahl der positiven rationalen Zahlen aus dem ersten und dem n-ten Einheitsintervall, also aus (0, 1] und (n, n+1], ausrechnen können, nicht einmal das Verhältnis aus (0, 1] und (1000, 1001]. Aber das ist sicher größer als 100.
Mir ist der Zusammenhang zwischen dieser Frage, und "Cantors berühmter
Folge" nicht klar. Was hat das eine mit dem anderen zu tun?
In Cantors Folge können wir das Verhältnis zwischen Mengen mathematisch analysieren, ohne auf die Bijektionsmethode zurückzugreifen. Damit können wir zeigen, dass die Bijektionsmethode mathematisch unhaltbar ist.
Post by Uwe Weiss
Die Kardinalitäten innerhalb der rationalen Zahlen von (0, 1] und (1000,
1001] sind identisch.
Und ebenso - und DAS ist beängstigend! - die von (0, 1] und (0, 2].
Das ist nicht beängstigend, sondern hängt mit der Bijektionsmethode zusammen.

Diese verwendet das Totschlagargument: Du kannst keinen nichtindizierten Bruch definieren. Also werden alle nummeriert.
Dass damit nur die potentiell unendliche Menge der definierbaren Elemente abgearbeitet ist, wird leider übersehen oder bewusst verdrängt. Für alle definierbaren Indizes gilt nämlich auch

∀n ∈ ℕ_def: |ℕ \ {1, 2, 3, ..., n}| = ℵo.

Du kannst keines finden, dass davon abweicht. Wieso aber sollten alle damit inditzierten Brüche alle Brüche sein?
Post by Uwe Weiss
Post by Ganzhinterseher
Kann wenigstens jemand die Bedeutung dieses Verhältnisses interpretieren?
Ja. Wie oben geschrieben: beängstigend!
Das Verhältnis wird nicht durch eine Bijektion erzeugt. Wollte man eine aufstellen, so würde natürlich jedem Bruch aus (0, 1) ein Bruch aus (1000, 1001) zur Seite gestellt werden können, eben weil die Bijektionsmethode nichts mit mathematischer Präzision zu tun hat.

Gruß, WM
Michael Klemm
2020-06-09 21:11:06 UTC
Permalink
Post by Ganzhinterseher
Post by Uwe Weiss
Post by Ganzhinterseher
1/1, 1/2, 2/1, 1/3, 3/1, 1/4, 2/3, 3/2, 4/1, 1/5, 5/1, 1/6, ... .
Bisher hat niemand den Grenzwert des Verhältnisses der Anzahl der positiven rationalen Zahlen aus dem ersten und dem n-ten Einheitsintervall, also aus (0, 1] und (n, n+1], ausrechnen können, nicht einmal das Verhältnis aus (0, 1] und (1000, 1001]. Aber das ist sicher größer als 100.
Mir ist der Zusammenhang zwischen dieser Frage, und "Cantors berühmter
Folge" nicht klar. Was hat das eine mit dem anderen zu tun?
In Cantors Folge können wir das Verhältnis zwischen Mengen mathematisch analysieren, ohne auf die Bijektionsmethode zurückzugreifen. Damit können wir zeigen, dass die Bijektionsmethode mathematisch unhaltbar ist.
Cantor benutzt keine Bijektion. Er definiert (verbal) die Ordnung
a/b <' c/d, genau wenn a+b < c+d oder (a+b = c+d und a/b > c/d).

Gruß
Michael
Post by Ganzhinterseher
Post by Uwe Weiss
Die Kardinalitäten innerhalb der rationalen Zahlen von (0, 1] und (1000,
1001] sind identisch.
Und ebenso - und DAS ist beängstigend! - die von (0, 1] und (0, 2].
Das ist nicht beängstigend, sondern hängt mit der Bijektionsmethode zusammen.
Diese verwendet das Totschlagargument: Du kannst keinen nichtindizierten Bruch definieren. Also werden alle nummeriert.
Dass damit nur die potentiell unendliche Menge der definierbaren Elemente abgearbeitet ist, wird leider übersehen oder bewusst verdrängt. Für alle definierbaren Indizes gilt nämlich auch
∀n ∈ ℕ_def: |ℕ \ {1, 2, 3, ..., n}| = ℵo.
Du kannst keines finden, dass davon abweicht. Wieso aber sollten alle damit inditzierten Brüche alle Brüche sein?
Post by Uwe Weiss
Post by Ganzhinterseher
Kann wenigstens jemand die Bedeutung dieses Verhältnisses interpretieren?
Ja. Wie oben geschrieben: beängstigend!
Das Verhältnis wird nicht durch eine Bijektion erzeugt. Wollte man eine aufstellen, so würde natürlich jedem Bruch aus (0, 1) ein Bruch aus (1000, 1001) zur Seite gestellt werden können, eben weil die Bijektionsmethode nichts mit mathematischer Präzision zu tun hat.
Gruß, WM
Ganzhinterseher
2020-06-10 16:52:53 UTC
Permalink
Post by Michael Klemm
Post by Ganzhinterseher
Post by Uwe Weiss
Post by Ganzhinterseher
1/1, 1/2, 2/1, 1/3, 3/1, 1/4, 2/3, 3/2, 4/1, 1/5, 5/1, 1/6, ... .
Bisher hat niemand den Grenzwert des Verhältnisses der Anzahl der positiven rationalen Zahlen aus dem ersten und dem n-ten Einheitsintervall, also aus (0, 1] und (n, n+1], ausrechnen können, nicht einmal das Verhältnis aus (0, 1] und (1000, 1001]. Aber das ist sicher größer als 100.
Mir ist der Zusammenhang zwischen dieser Frage, und "Cantors berühmter
Folge" nicht klar. Was hat das eine mit dem anderen zu tun?
In Cantors Folge können wir das Verhältnis zwischen Mengen mathematisch analysieren, ohne auf die Bijektionsmethode zurückzugreifen. Damit können wir zeigen, dass die Bijektionsmethode mathematisch unhaltbar ist.
Cantor benutzt keine Bijektion.
Doch.

Das Gesetz dieser Reihe ist ein höchst einfaches: Sie sehen, dass die Reihe nach gewissen Abschnitten fortschreitet, von denen jeder zwischen zwei ; ; eingeschlossen ist. Der erste Abschnitt enthält (2) = 1, der zweite (3) = 2, der (n - 1)-te Abschnitt (n) Zahlen, wo (n) die Anzahl aller relativen Primzahlen zu n, die kleiner als n sind, bestimmt.
Innerhalb des (n - 1)ten Abschnittes bilden die Zähler der rationalen Zahlen die aufsteigende Reihe der (n) Zahlen rel. prim zu n und kleiner als n, die Nenner die absteigende Reihe derselben (n) Zahlen.
Die so definirte unendliche Reihe hat nun das merkwürdige an sich, sämmtliche positiven rationalen Zahlen und jede von ihnen nur einmal an einer bestimmten Stelle zu enthalten.

Das ist das Kennzeichen einer Bijektion mit |N.

Nur sind eben nicht sämtlich positiven rationalen Zahlen darin enthalten, sondern nur solche, deren Zähler und Nenner summiert eine Zahl n ergeben, die um unendliche viele von der Vollständigkeit entfernt ist:

∀n ∈ Cantors Sämtlichkeit: |ω| - n = |ω|

Die Behauptung ist also klar widerlegt.

Gruß, WM
Michael Klemm
2020-06-10 19:36:13 UTC
Permalink
Post by Ganzhinterseher
Post by Michael Klemm
Post by Ganzhinterseher
Post by Uwe Weiss
Post by Ganzhinterseher
1/1, 1/2, 2/1, 1/3, 3/1, 1/4, 2/3, 3/2, 4/1, 1/5, 5/1, 1/6, ... .
Bisher hat niemand den Grenzwert des Verhältnisses der Anzahl der positiven rationalen Zahlen aus dem ersten und dem n-ten Einheitsintervall, also aus (0, 1] und (n, n+1], ausrechnen können, nicht einmal das Verhältnis aus (0, 1] und (1000, 1001]. Aber das ist sicher größer als 100.
Mir ist der Zusammenhang zwischen dieser Frage, und "Cantors berühmter
Folge" nicht klar. Was hat das eine mit dem anderen zu tun?
In Cantors Folge können wir das Verhältnis zwischen Mengen mathematisch analysieren, ohne auf die Bijektionsmethode zurückzugreifen. Damit können wir zeigen, dass die Bijektionsmethode mathematisch unhaltbar ist.
Cantor benutzt keine Bijektion.
Doch.
Das Gesetz dieser Reihe ist ein höchst einfaches: Sie sehen, dass die Reihe nach gewissen Abschnitten fortschreitet, von denen jeder zwischen zwei ; ; eingeschlossen ist. Der erste Abschnitt enthält (2) = 1, der zweite (3) = 2, der (n - 1)-te Abschnitt (n) Zahlen, wo (n) die Anzahl aller relativen Primzahlen zu n, die kleiner als n sind, bestimmt.
Innerhalb des (n - 1)ten Abschnittes bilden die Zähler der rationalen Zahlen die aufsteigende Reihe der (n) Zahlen rel. prim zu n und kleiner als n, die Nenner die absteigende Reihe derselben (n) Zahlen.
Die so definirte unendliche Reihe hat nun das merkwürdige an sich, sämmtliche positiven rationalen Zahlen und jede von ihnen nur einmal an einer bestimmten Stelle zu enthalten.
Das ist das Kennzeichen einer Bijektion mit |N.
Wo steht da etwas von |N oder einer Bijektion? Die natürliche Ordnungsrelation der rationalen Zahlen muss ja bei einem systematischen Aufbau ebenfalls definiert und ihre Eigenschaften beschrieben werden. Da kann man dann auch feststellen, dass jede rationale Zahl nur einmal an einer bestimmten Stelle der "Reihe" vorkommt.

Gruß
Michael
Post by Ganzhinterseher
∀n ∈ Cantors Sämtlichkeit: |ω| - n = |ω|
Die Behauptung ist also klar widerlegt.
Gruß, WM
Ganzhinterseher
2020-06-11 13:50:39 UTC
Permalink
Post by Michael Klemm
Post by Ganzhinterseher
Post by Michael Klemm
Post by Ganzhinterseher
Post by Uwe Weiss
Post by Ganzhinterseher
1/1, 1/2, 2/1, 1/3, 3/1, 1/4, 2/3, 3/2, 4/1, 1/5, 5/1, 1/6, ... .
Bisher hat niemand den Grenzwert des Verhältnisses der Anzahl der positiven rationalen Zahlen aus dem ersten und dem n-ten Einheitsintervall, also aus (0, 1] und (n, n+1], ausrechnen können, nicht einmal das Verhältnis aus (0, 1] und (1000, 1001]. Aber das ist sicher größer als 100.
Mir ist der Zusammenhang zwischen dieser Frage, und "Cantors berühmter
Folge" nicht klar. Was hat das eine mit dem anderen zu tun?
In Cantors Folge können wir das Verhältnis zwischen Mengen mathematisch analysieren, ohne auf die Bijektionsmethode zurückzugreifen. Damit können wir zeigen, dass die Bijektionsmethode mathematisch unhaltbar ist.
Cantor benutzt keine Bijektion.
Doch.
Das Gesetz dieser Reihe ist ein höchst einfaches: Sie sehen, dass die Reihe nach gewissen Abschnitten fortschreitet, von denen jeder zwischen zwei ; ; eingeschlossen ist. Der erste Abschnitt enthält (2) = 1, der zweite (3) = 2, der (n - 1)-te Abschnitt (n) Zahlen, wo (n) die Anzahl aller relativen Primzahlen zu n, die kleiner als n sind, bestimmt.
Innerhalb des (n - 1)ten Abschnittes bilden die Zähler der rationalen Zahlen die aufsteigende Reihe der (n) Zahlen rel. prim zu n und kleiner als n, die Nenner die absteigende Reihe derselben (n) Zahlen.
Die so definirte unendliche Reihe hat nun das merkwürdige an sich, sämmtliche positiven rationalen Zahlen und jede von ihnen nur einmal an einer bestimmten Stelle zu enthalten.
Das ist das Kennzeichen einer Bijektion mit |N.
Wo steht da etwas von |N oder einer Bijektion?
Direkt vor Deiner Nase. (Hinweis: Cantor bezeichnete das, was wir heute eine Folge nennen, noch als Reihe.)

Gruß, WM
Michael Klemm
2020-06-11 15:18:40 UTC
Permalink
Post by Ganzhinterseher
Post by Michael Klemm
Post by Ganzhinterseher
Post by Michael Klemm
Post by Ganzhinterseher
Post by Uwe Weiss
Post by Ganzhinterseher
1/1, 1/2, 2/1, 1/3, 3/1, 1/4, 2/3, 3/2, 4/1, 1/5, 5/1, 1/6, ... .
Bisher hat niemand den Grenzwert des Verhältnisses der Anzahl der positiven rationalen Zahlen aus dem ersten und dem n-ten Einheitsintervall, also aus (0, 1] und (n, n+1], ausrechnen können, nicht einmal das Verhältnis aus (0, 1] und (1000, 1001]. Aber das ist sicher größer als 100.
Mir ist der Zusammenhang zwischen dieser Frage, und "Cantors berühmter
Folge" nicht klar. Was hat das eine mit dem anderen zu tun?
In Cantors Folge können wir das Verhältnis zwischen Mengen mathematisch analysieren, ohne auf die Bijektionsmethode zurückzugreifen. Damit können wir zeigen, dass die Bijektionsmethode mathematisch unhaltbar ist.
Cantor benutzt keine Bijektion.
Doch.
Das Gesetz dieser Reihe ist ein höchst einfaches: Sie sehen, dass die Reihe nach gewissen Abschnitten fortschreitet, von denen jeder zwischen zwei ; ; eingeschlossen ist. Der erste Abschnitt enthält (2) = 1, der zweite (3) = 2, der (n - 1)-te Abschnitt (n) Zahlen, wo (n) die Anzahl aller relativen Primzahlen zu n, die kleiner als n sind, bestimmt.
Innerhalb des (n - 1)ten Abschnittes bilden die Zähler der rationalen Zahlen die aufsteigende Reihe der (n) Zahlen rel. prim zu n und kleiner als n, die Nenner die absteigende Reihe derselben (n) Zahlen.
Die so definirte unendliche Reihe hat nun das merkwürdige an sich, sämmtliche positiven rationalen Zahlen und jede von ihnen nur einmal an einer bestimmten Stelle zu enthalten.
Das ist das Kennzeichen einer Bijektion mit |N.
Wo steht da etwas von |N oder einer Bijektion?
Direkt vor Deiner Nase. (Hinweis: Cantor bezeichnete das, was wir heute eine Folge nennen, noch als Reihe.)
Gruß, WM
Es geht hier aber um die Frage, ob Cantor den Begriff "Reihe" auch für lineare Ordnungen wie die natürliche Ordnung oder die Wohlordnung der rationalen Zahlen verwendet.

Gruß
Michael
Ganzhinterseher
2020-06-11 18:41:09 UTC
Permalink
Post by Michael Klemm
Post by Ganzhinterseher
Post by Michael Klemm
Post by Ganzhinterseher
Das Gesetz dieser Reihe ist ein höchst einfaches: Sie sehen, dass die Reihe nach gewissen Abschnitten fortschreitet, von denen jeder zwischen zwei ; ; eingeschlossen ist. Der erste Abschnitt enthält (2) = 1, der zweite (3) = 2, der (n - 1)-te Abschnitt (n) Zahlen, wo (n) die Anzahl aller relativen Primzahlen zu n, die kleiner als n sind, bestimmt.
Wo steht da etwas von |N oder einer Bijektion?
Direkt vor Deiner Nase. (Hinweis: Cantor bezeichnete das, was wir heute eine Folge nennen, noch als Reihe.)
Es geht hier aber um die Frage, ob Cantor den Begriff "Reihe" auch für lineare Ordnungen wie die natürliche Ordnung oder die Wohlordnung der rationalen Zahlen verwendet.
Hebe Deinen Blick ein wenig. Oben wird das beantwortet.

Gruß, WM
Michael Klemm
2020-06-12 12:04:07 UTC
Permalink
Post by Ganzhinterseher
Post by Michael Klemm
Post by Ganzhinterseher
Post by Michael Klemm
Post by Ganzhinterseher
Das Gesetz dieser Reihe ist ein höchst einfaches: Sie sehen, dass die Reihe nach gewissen Abschnitten fortschreitet, von denen jeder zwischen zwei ; ; eingeschlossen ist. Der erste Abschnitt enthält (2) = 1, der zweite (3) = 2, der (n - 1)-te Abschnitt (n) Zahlen, wo (n) die Anzahl aller relativen Primzahlen zu n, die kleiner als n sind, bestimmt.
Wo steht da etwas von |N oder einer Bijektion?
Direkt vor Deiner Nase. (Hinweis: Cantor bezeichnete das, was wir heute eine Folge nennen, noch als Reihe.)
Es geht hier aber um die Frage, ob Cantor den Begriff "Reihe" auch für lineare Ordnungen wie die natürliche Ordnung oder die Wohlordnung der rationalen Zahlen verwendet.
Hebe Deinen Blick ein wenig. Oben wird das beantwortet.
Gruß, WM
Cantor bezeichnet Mengen mit "Inbegriff (griechischer Buchstabe)". Der griechische Buchstabe ist dabei eine Variable und keine fester Index aus der "Reihe" 1,2,3,4,5,6,7,... (bis der Lehrer pitsch, patsch macht).

Gruß
Michael
Ganzhinterseher
2020-06-10 17:03:43 UTC
Permalink
Post by Uwe Weiss
Und ebenso - und DAS ist beängstigend! - die von (0, 1] und (0, 2].
Das wäre nicht allein beängstigend, sondern falsch, wenn wirklich alle Zahlen erfasst würden. Aber Cantors Bijektion erfasst nur einen kleinen, nicht nennenswerten Bruchteil der aktual unendlichen Mengen. Durch die Identität der Kardinalzahlen von (0, 1] und (-oo, oo) wird dies offenbar. Ein anderer Beweis ist der, dass für alle von Cantor erfassten natürlichen Zahlen gilt:

∀n ∈ ℕ_erfasst: |ω| - n = |ω|.

Es fehlen also in jedem Falle unendlich viele. Eigentlich nicht schwer zu verstehen.

Gruß, WM
Me
2020-06-10 17:30:13 UTC
Permalink
Post by Ganzhinterseher
Es fehlen also in jedem Falle unendlich viele. Eigentlich nicht schwer zu verstehen.
Mückenheim, können Sie Ihren saudummen Scheißdreck nicht wo anders abseichen?
jvr
2020-06-10 18:51:33 UTC
Permalink
Post by Ganzhinterseher
Post by Uwe Weiss
Und ebenso - und DAS ist beängstigend! - die von (0, 1] und (0, 2].
∀n ∈ ℕ_erfasst: |ω| - n = |ω|.
Es fehlen also in jedem Falle unendlich viele. Eigentlich nicht schwer zu verstehen.
Gruß, WM
Dummheit ist nicht «wenig wissen», auch nicht «wenig wissen wollen», Dummheit ist «glauben, genug zu wissen».

Konfuzius
Ganzhinterseher
2020-06-11 13:47:49 UTC
Permalink
Post by jvr
Post by Ganzhinterseher
∀n ∈ ℕ_erfasst: |ω| - n = |ω|.
Es fehlen also in jedem Falle unendlich viele. Eigentlich nicht schwer zu verstehen.
Dummheit ist nicht «wenig wissen», auch nicht «wenig wissen wollen», Dummheit ist «glauben, genug zu wissen».
Konfuzius
Das war ein kluger Mann! Mit Pythagoras und Buddha hat er das erste wissenschaftliche Erwachen der Menschheit eingeleitet. Er kannte zwar die Matheologie noch nicht, aber sein Spruch trifft genau auf sie zu.

Denn wer glaubt oder behauptet oder sogar glaubt und behauptet, dass er alle Zahlen erfasst hat, obwohl die nicht erfasste Menge immer unendlich viele größer ist als die erfasste, ist dumm.

Gruß, WM
Juergen Ilse
2020-06-10 22:52:06 UTC
Permalink
Hallo,
Post by Ganzhinterseher
Post by Uwe Weiss
Und ebenso - und DAS ist beängstigend! - die von (0, 1] und (0, 2].
Das wäre nicht allein beängstigend, sondern falsch, wenn wirklich alle Zahlen erfasst würden. Aber Cantors Bijektion erfasst nur einen kleinen, nicht nennenswerten Bruchteil der aktual unendlichen Mengen. Durch die Identität der Kardinalzahlen von (0, 1] und (-oo, oo) wird dies offenbar.
Durch IHRE Argumentation wird nuroffenbar, dass SIE zu daemlich sind, um
unendlichkeit in der Mathematik zu verstehen.

Tschuess,
Juergen Ilse (***@usenet-verwaltung.de)
Ganzhinterseher
2020-06-11 14:34:15 UTC
Permalink
Post by Juergen Ilse
Hallo,
Post by Ganzhinterseher
Post by Uwe Weiss
Und ebenso - und DAS ist beängstigend! - die von (0, 1] und (0, 2].
Das wäre nicht allein beängstigend, sondern falsch, wenn wirklich alle Zahlen erfasst würden. Aber Cantors Bijektion erfasst nur einen kleinen, nicht nennenswerten Bruchteil der aktual unendlichen Mengen. Durch die Identität der Kardinalzahlen von (0, 1] und (-oo, oo) wird dies offenbar.
Durch IHRE Argumentation wird nuroffenbar, dass SIE zu daemlich sind, um
unendlichkeit in der Mathematik zu verstehen.
Mit dem Verstehen ist das so eine Sache. Es wird besonders von Kirchen und Matheologen gefordert. Leider muss man in beiden Fällen den Verstand ausschalten, mehr oder weniger stark.

Die leibliche Himmelfahrt Mariens ist schon starker Tobak, aber das ist doch noch gar nichts im Vergleich zu der Behauptung, dass in einem Quadrat genau so viele Brüche ihr Unterkommen finden wie auf einer Seite, oder dass auf dem Radius eines Quarks genau so viele rationale Punkte sitzen wie im gesamten Universum.

Dass Cantor es geschafft hat, die Mathematiker dermaßen zu bluffen, das ist seine eigentliche geniale Leistung. Und wodurch? Allein durch die ungeprüfte Behauptung, dass er bis omega zählen kann, so dass nichts übrig bleibt, obwohl er ebenfalls, an etwas versteckter Stelle zugibt, dass für alle seine Zählzahlen gilt: ∀n ∈ ℕ: |ω| - n = |ω|, er also nicht bis omega zählen kann.

Gruß, WM
Loading...